Download as pdf or txt
Download as pdf or txt
You are on page 1of 136

MCQ BANK

MCQs by topic
Paediatrics
NOV 2007

1. The following is SNHL syndrome


a. Pendred’s syndrome
b. Usher
c. Alport’s
d. Treacher Collin’s
e. Crouzon TTTFF

2. Pierre robin’s syndrome feature


a. Microcephaly or small head
b. Mandibular bone deformity
c. Cochlear deformity
FTF

3. The following are congenital cause of hearing loss


a. Congenital cholesteatoma
b. Otosclerosis
c. Otosclerosis is familial disease
d. Endartheritis
e. Cogan’s syndrome
TTTFT

MAY 2008

1. In inherited hearing loss


a. Mitchel aplasia is a complete aplasia
b. Mondini only basal coil is present
c. Alexander involve membranous pt of cochlea
d. Sheibe cochleosaccular dysplasia
e. Wardenburg is a pathological lesion is common congenital SNHL
TFTTT

2. In paediatric tracheostomy
a. Easily performed
b. Done under LA
c. Window Is created in trachea
d. Is commonly done in ELTB
e. Is preferred than intubation
FFFFF

3. In paediatric larynx
a. The superior boarder at C1
b. The infantile larynx is 1/3 of adult larynx
c. In adolescence subglottic is narrowest
d. The subglottic diameter in infantile is 4 mm
e. 1mm narrowing in lumen cause 75% of obstruction
TTFTT

4. Features of Pierre Robin


a. Hypoplasia mandible
b. Acrocephaly
c. Syndactyly
d. Absence of middle ear cleft
e. Abnormality in modiolus of cochlea
TFFFF
14. Differential diagnoses in congenital nasal mass
a. Dermoid cyst
b. Lacrimal duct cyst
c. Neurofibroma
d. Sebaceous cyst
e. Hemangioma
TFTFT

NOV 2008

1. Regarding cochlear implantation


a. It is not justified in patients younger than 18 month
b. Its rehabilitation stresses on oral communication
c. Cochlear aplasia is absolute contraindication
d. It works by stimulating the neurons of the spiral ganglion
e. The stimulating electrode is implanted into scala vestibuli
FTTTF

2. The following presents with conductive deafness at birth


a. Down syndrome
b. Treacher Collin syndrome
c. Apert syndrome
d. Refsum disease
e. Alport syndrome
TTTFF

3. Regarding mucosal abnormalities


a. Kartagener syndrome is related to abnormal mucous
b. Sinusitis in Young syndrome results from abnormal ciliary

activity
c. Children with cystic fibrosis disease has immotile cilia
d. Chest X-ray is useful in diagnosis of Kartagener syndrome
e. Sweat test is useful in children presenting with nasal polyps

FFFTT

4. Congenital pinna abnormalities can be caused by


a. Warfarin
b. Methotrexate
c. Alcohol
d. Thalidomide
e. Benzodiazepines
TTTTF

5. The following are causes of congenital hearing loss with corresponding


pathology
a. Michel’s: complete agenesis
b. Mondini’s : incomplete bony and membranous labyrinth
c. Sciebe : membranous cochlear aplasia
d. Alexander : cochleosaccular aplasia
e. Alport’s : cochlear aplasia
TTFFF

6. The following are causes of syndromic sensorineural hearing loss


a. Cockayne’s syndrome
b. Cogans syndrome
c. Wildervanck’s syndrome
d. Potter’s syndrome
e. Marfans syndrome
TTTTT
7. The following are true of laryngomalacia
a. Causes expiratory stridor
b. Occurs predominantly in low birth weight babies
c. Indrawing of the epiglottis indicates severity
d. Majority are treated by incision of the aryepiglottic fold
e. When persists after the age of 4 years will require re-evaluation
of the diagnosis
FFTFT

8. Paediatric tracheostomy
a. Is frequently required for acute laryngotracheobronchitis
b. Is preferable to intubation in an emergency
c. Is complicated by psychological dependence
d. Involves cutting a slit in 1st and 3rd tracheal rings
e. Is easier performed in children because of their shorter neck
FFTFF

9. The following are true of the stridor in infants


a. The commonest site of a web is at the anterior commissure
b. These infant are completely assessed under general anaesthesia
c. The stridor is commonly associated with feeding difficulties
d. Laryngeal cysts causing stridor are commonly found in the
supraglottis
e. Bilateral abductor palsy is best treated early with laser
cordectomy
TFTTF

10. The following statement regarding tracheal stenosis are true


a. It is most commonly caused blunt trauma
b. Cotton grade 3 is complete stenosis
c. The stridor is commonly associated with feeding difficulties
d. Laryngeal cysts causing stridor are commonly found in the
supraglottis
e. Bilateral abductor palsy is best treated early with laser
cordectomy
FFFTF

11. Hand-Schuller-Christian disease is


a. Common in young adults
b. Frequently associated with diabetes insipidus
c. The most benign form of histiocytosis X
d. Usually found in a single focus
e. Invariably fatal
FTFFF
May 2012
1. Micrognathia is associated with
a. ( ) Apert syndrome
b. ( ) Pierre-robin
c. ( ) Treacher collin
d. ( ) Cri-du-chat syndrome
e. ( ) Crouzon’s syndrome

2. Syndromic SNHL
a. ( ) Cogan’s syndrome
b. ( ) Down’s syndrome
c. ( ) Potter’s syndrome
d. ( ) Marfan’s syndrome

3. Laryngotracheosophageal cleft
a. ( ) Present with husky cry
b. ( ) The phase of interruption of tracheoesophageal septum
determines the length of the cleft
c. ( ) It occurs in Pallister Hall syndrome
d. ( ) Type 1 involves cricoid
e. ( ) Type 1 repair is best approached through a lateral
pharyngotomy approach

4. Better outcome of CI in children is determine by


a. ( ) Sudden onset SNHL
b. ( ) Older child
c. ( ) Longer duration of Ci wear
d. ( ) Aural rehabilitation

5. Parotid tumour in children


a. ( ) In children, commonest neoplasm is mucoepidermoid
carcinoma
b. ( ) Oncocytoma is a benign tumour
c. ( ) Acinic cell ca has risk of neck mets
d. ( ) Adenoid cystic is the commonest malignant tumour in
submandibular gland

6. Syndromes associated with facial palsy


a. ( ) Goldenhar syndrome
b. ( ) Mobius syndrome
c. ( ) Di George syndrome
d. ( ) CHARGE association
e. ( ) Fascioscapulohumeral muscular dystrophy
MAY 2013

1. Macroglossia causes
a. ( ) Beck width Wiedemann syndrome
b. ( ) Pierre robin
c. ( ) Amyloidosis
d. ( ) Neurofibroma
e. ( ) Apert
f. ( ) Treacher Collin syndrome

2. Cystic hygroma
a. ( ) Is a type of venous malformation
b. ( ) Picibanil Is a good sclerosant
c. ( ) Will spontaneously regress at 7-8 yo

3. Regarding congenital vascular anomalies


a. ( ) Subglottic hemangioma is diagnosed with tissue biopsy
b. ( ) Haemangioma occur at birth
c. ( ) Port wine stain is capillary AVM

4. Features of Down syndrome


a. ( ) Sialorrhea
b. ( ) Brachycephaly
c. ( ) Narrow trachea
d. ( ) Mondini dysplasia

5. Sarcoma
a. ( ) Commonest soft tissue malignancy in children
b. ( ) Less than 10yo is uncommon
c. ( ) Head and neck sarcoma in adults are the best treated with
chemotherapy
May 2014

1.
Laryngomalacia
a. ( ) Is caused by immature development of laryngeal cartilage
b. ( ) Is caused by incoordination of laryngeal-pharyngeal
structure
c. ( ) There Is no need examine tracheobronchial area if the child
is planned for op
2. Stridor
a. ( ) Subglottic hemangioma present at birth
b. ( ) Subglottic stenosis is usually treated endoscopically before
external surgery
c. ( ) Anterior cricoid split is indication in congenital case
d. ( ) Bilateral vocal cord palsy present with high pitched cry
e. ( ) Tracheostomy should be done in unilateral vocal cord palsy

3. Kimura disease
a. ( ) Is also termed as a eosinophilic angiolymphoid lesion
b. ( ) Has malignant potential
c. ( ) Has subcutaneous tissue involvement
d. ( ) Clinically mimic lymphoma
e. ( ) Has positive Ig G

4. True regarding hearing in children


a. ( ) Newborn able to differentiate sound frequency and
intensity
b. ( ) Able to differentiate phemomes
c. ( ) Hearing can only be tested at 1 years old

5. Foreign body
a. ( ) Can be excluded if x-ray showed negative findings
b. ( ) Non- organic can stay asymptomatic in lower airway for
long duration
c. ( ) Associated with bronchiectasis lung
d. ( ) Related to emphysematous lung finding
May 2015
1. Regarding tympanometry, 2years old boy with history of grommets
inserted bilaterally, presented with right central perforation with
recurrent of left glue ear
a. ( ) Left tymp type C
b. ( ) Right tympanoplasty is required in future
c. ( ) Type B on right with normal volume
d. ( ) Bilateral hearing aids are beneficial
e. ( ) OAE show bilateral positive response

2. Tracheomalacia
a. ( ) Diffuse more common than localized
b. ( ) Decreased AP diameter
c. ( ) Caused by compression of innominate artery
d. ( ) Caused expiratory stridor
e. ( ) Can resolved spontaneously

3. Treacher-collins
a. ( ) Autosomal dominant
b. ( ) Micrognathia
c. ( ) Canal atresia
d. ( ) Choanal atresia

4. Choanal atresia
a. ( ) Bony 90%, membranous 10%
b. ( ) Bony of posterior wall of maxilla
c. ( ) Occurs in Treacher -Collin syndrome
d. ( ) Nasal secretion is a symptom
e. ( ) Low APGAR score

6. Risk of ASOM
a. ( ) High social status
b. ( ) Bottle feeding
c. ( ) Attendance to daycare facilities
d. ( ) Premature
e. ( ) Aborigin indigenous
Nov 2015
1. Subglottic hemangioma
a. ( ) Propranolol is a treatment
b. ( ) Steroid is a treatment
c. ( ) Born with stridor
d. ( ) Associated with subcutaneous hemangioma
2. Axial CT scan of choanal atresia
a. ( ) Airway of less than 3mm at pterygoid plate
b. ( ) Thick vomer
c. ( ) Inward bowing of maxillary spine
d. ( ) Narrow pyriform aperture
e. ( ) Medial bowing posterior maxillary sinus

3. Cystic hygroma
a. ( ) Ill defined CT
b. ( ) Cholesterol crystal
c. ( ) Neuro element detected
d. ( ) Line by endothelial cell

4. Causes of laryngotracheal stenosis


a. ( ) Tracheomalacia
b. ( ) Subglottic hemangioma
c. ( ) Cutaneous patchyderma

5. Stridor in children
a. ( ) Glottis web present weak cry
b. ( ) Supraglottic cyst is differential diagnosis

6. Acute otitis media


a. ( ) Common in children
b. ( ) Reduce in breast feeding
c. ( ) First sign is bulging TM
d. ( ) Usually have spontaneous healing
e. ( ) Commonest organism is klebsiella pneumonia

7. Universal hearing screening


a. ( ) Done in all baby
b. ( ) OAE is frequency specific
c. ( ) ABR min 20 dB HL
d. ( ) Mild hearing loss usually missed
e. ( ) Repeat after 3 months if fail

8. Cochlear implant
a. ( ) As early as possible in prelingual
b. ( ) Hearing aid is pre-requirement
c. ( ) Require mapping pre- op
MAY 2016

1. Trachea- oesophageal fistula is associated with


a. ( ) Vertebral anomalies
b. ( ) Anorectal malformation
c. ( ) Oesophageal atresia
d. ( ) Thyroglossal cysts
e. ( ) Renal dysplasia

2. OAE in right ear with moderate CHL and left normal ear
a. ( ) Left pass
b. ( ) Right pass
c. ( ) Both pass
d. ( ) Left refer
e. ( ) Right refer

3. Screening test for neonates hearing


a. ( ) ASSR
b. ( ) ABR
c. ( ) AABR
d. ( ) Tympanometry

4. Universal neonatal screening


a. ( ) All tested before discharged
b. ( ) Repeat test if abnormal
c. ( ) 1-4kHz used

5. Causes of cleft palate


a. ( ) Treacher Collin
b. ( ) Crouzon
c. ( ) Marfan syndrome

6. Tracheoesophageal fistula associated with


a. ( ) Vertebral anomalies
b. ( ) Anorectal anomalies
c. ( ) Oesophageal atresia
d. ( ) Renal dysplasia
NOV 2016

1. Regarding cochlear implant


a. ( ) It is indicated as early as 1 year old
b. ( ) Multichannel send different electrical signals to different
sites in the cochlea
c. ( ) Patient with no residual hearing should not have the
implant
d. ( ) Pre-evaluation counselling play a minimal role in suitable
candidates for implant
e. ( ) Meningitis is unusual complication
f. ( ) Aural rehabilitation include leaning sign language

2. Following are conductive hearing loss at birth


a. ( ) Down syndrome
b. ( ) Treacher Collin
c. ( ) Apert syndrome
d. ( ) Refsum syndrome
e. ( ) Alport syndrome

3. Regarding congenital nasal pyriform aperture stenosis


a. ( ) It results from bony overgrowth of nasal process of palatine
bone
b. ( ) The height of the nasal cavity is not altered
c. ( ) It is associated with holoprosenephaly
d. ( ) The aperture width is less than 8mm
e. ( ) Surgical correction is the treatment of choice

4. Tracheoesophageal fistula
a. ( ) It is associated with CHARGE syndrome
b. ( ) Bronchoscopy is mandatory
c. ( ) CXR showed air bubble in distal to fistula
d. ( ) The commonest is the H type
e. ( ) Excessive drooling is a presentation

5. Clinical features of congenital stridor includes


a. ( ) Recurrent fever
b. ( ) Aspiration
c. ( ) Abnormal cry
d. ( ) Cyclical apnea
e. ( ) Recurrent croup

6. Predisposing factors for paediatric post intubation laryngeal stenosis


include:
a. ( ) Size of the endotracheal tube
b. ( ) Normal laryngeal anatomy
c. ( ) Undiagnosed congenital airway lesions
d. ( ) Gastro-esophageal reflux
e. ( ) Anchoring of the endotracheal tube
Nov 2017

1. Child on prolonged tracheostomy


a. ( ) Endoscopy before decannulation
b. ( ) Metal tube to prevent stenosis
c. ( ) Suprastomal granulation treated with larger tube
d. ( ) Contraindicated in cystic fibrosis due to fear of mucus
plug
e. ( ) Early tracheocutaneous fistula is needed

2. To assess hearing in neonates


a. ( ) BSER
b. ( ) OAE
c. ( ) Impedance audiometry
d. ( ) Behavioural audiometry
e. ( ) Distraction test/VRA

3. Juvenile recurrent parotitis


a. ( ) A/w sialolith
b. ( ) Bilateral
c. ( ) Diagnosed with sialography
d. ( ) Treat with ductal flushing
e. ( ) Resolves spontaneously after puberty

4. Tracheomalacia
a. ( ) 50% associated with bronchomalacia
b. ( ) 50% a/w LPR
c. ( ) Present with wheezing
d. ( ) Artopexy is treatment of choice
e. ( ) Secondary tracheomalacia caused by prolonged
intubation

5. Laryngeal web
a. ( ) A/w DiGeorge syndrome
b. ( ) Causes SGS
c. ( ) A/w Cardiac anomalies
d. ( ) CHAOS syndrome worst prognosis
e. ( ) Cohen type 2 a/w 30% glottic stenosis

6. Paed head and neck tumour


a. ( ) Lymphoma commonest
b. ( ) Rhabdomyosarcoma is most common soft tissue tumor
c. ( ) Thyroid nodule more commonly malignant compared
to adult
d. ( ) WHO type 1 NPC more commonly seen
e. ( ) Immature teratoma is benign tumour
NOV 2018

1. The indication for ventilation tube in children with middle ear effusion
includes
a. ( ) acute otitis media with facial nerve palsy
b. ( ) cleft lip
c. ( ) documented hearing difficulties
d. ( ) recurrent acute otitis media
e. ( ) unilateral effusion for more than three months

2. In branchial cyst,
a. ( ) malignant transformation is common.
b. ( ) metastatic lymphadenopathy is one of the differential
diagnosis.
c. ( ) most branchial cysts are associated with internal openings.
d. ( ) presence of cholesterol crystals on cytology is a feature
e. ( ) the peak of onset is during childhood.

3. In laryngomalacia,
a. ( ) endoscopic assessment is necessary in all cases.
b. ( ) neck X-ray is required in its diagnosis.
c. ( ) Pectus excavatum resolves completely after
supraglottoplasty.
d. ( ) stridor occurs immediately after birth.
e. ( ) the stridor improves with chin lift.

4. Flexible nasopharyngo-laryngoscopy in children


a. ( ) causes laryngo spasm.
b. ( ) diagnose subglottic stenosis.
c. ( ) is in office procedure.
d. ( ) is performed to diagnose congenital vocal cord palsy.
e. ( ) is used to remove foreign body in the oropharynx.

5. Regarding Computed Tomography scan findings of neck swelling in children


a. ( ) Second branchial cleft cyst is well circumscribed.
b. ( ) Lymphatic malformation (cystic hygroma) is a hyper-
attenuating mass.
c. ( ) Cervical teratoma shows rim enhancement.
d. ( ) Thyroglossal duct cyst is a homogeneous mass.
e. ( ) Infectious mononucleosis presents with hypodense lymph
nodes.

6. Hemangioma in infancy
a. ( ) commonly presents at birth.
b. ( ) is associated with PHACE syndrome.
c. ( ) is confirmed by detection of glucose transporter 1 (GLUT-1).
d. ( ) is non involute.
e. ( ) predominantly occurs in females.

7. Congenital anomalies presenting with micrognathia include


a. ( ) Aport syndrome
b. ( ) Crouzon syndrome
c. ( ) Goldenhaar syndrome
d. ( ) Pierre Robin sequence
e. ( ) Treacher Collins syndrome
8. In the assessment of paediatric swallowing function,
a. ( ) “white out” phase during fiberoptic endoscopic evaluation
of swallowing (FEES) occurs due to hyolaryngeal elevation.
b. ( ) bed ridden children is best assessed with videofluoroscopic
swallow study (VFSS).
c. ( ) FEES is contraindicated in neonates.
d. ( ) patient handling of secretion is best seen with FEES.
e. ( ) VFSS is used to assess incomplete opening of the upper
esophageal sphincter.

NOV 2019
1. M&G
a. ( ) Incision made at Posterior inferior quadrant
b. ( ) Main aim is to improve hearing
c. ( ) Injury of chorda tympani
d. ( ) Treatment for cholesterol granuloma
e. ( ) Can leave for 1 year

2. Paeds trache
a. ( ) Size 3.5mm in term
b. ( ) Cuff reduces air leak
c. ( ) Pneumothorax is common
d. ( ) Atlantoaxial in downs
e. ( ) Window made in anterior wall

3. Hearing impairment in children


a. ( ) Progress in 2yo in syphilis
b. ( ) Usher present at 2nd decade
c. ( ) Mondini is fluctuating loss
d. ( ) Reyes syndrome

4. Vascular malformation
a. ( ) Presents at birth
b. ( ) AVM treated with propranolol
c. ( ) FNAC is required to diagnose lymphatic malformation

5. Choanal atresia
a. ( ) Vomer thickening
b. ( ) Measurement at pterygoid plates is less than 3mm
c. ( ) Anterior bowing of posterior wall max sinus
d. ( ) Central megaincisor
e. ( ) Pyriform aperture stenosis

6. Bilateral VC palsy at birth. When to do trache, presenting symptom


dysphonia.
a. ( ) Surgical intervention by 12 months.
b. ( ) Most in adduction position
c. ( ) Tracheostomy is needed
d. ( ) 12 months old require intervention
e. ( ) Dysphonia is the presentation
ADDITIONAL SET PAEDIATRICS

1. In juvenile laryngeal papillomatosis


a. Hoarseness is the commonest presenting symptom
b. Vocal cords and ventricular bands are commonly involved
c. A surgical procedure with venturi jet ventilation may spread
the disease distally
d. α interferon is the recommended adjuvant therapy
e. Death usually follows lung parenchymal involvement

2. The following are true of a child with long term tracheostomy


a. Endoscopy prior to decannulation is essential
b. There may be predisposition to recurrent chest infections
c. A suprastomal granulation results in decannulation difficulties
d. Primary surgical closure may result in surgical emphysema
e. A plastic tracheostomy tube is preferred in children

3. Sleep apnoea in children


a. Is a potential cause of death in infant death syndrome
b. Is associated with failure to thrive and feeding difficulties in
infants
c. Bradycardia is a sign of minor importance
d. Due to retropharyngeal swelling is best shown on an
occipitomental view x-ray
e. Is usually caused by enlargement of lingual tonsil
NOV 2020

1. ENT issues in Down’s syndrome


a. Sensorineural hearing loss
b. Tracheal stenosis
c. Obstructive sleep apnoea
d. Midfacial hypoplasia
e. Allergic rhinitis

2. Paeds OSA
a. Pierre-Robin
b. Related to breathing center pathology?
c. Uncommon snoring
d. AHI 4 is normal

3. Lymphatic malformation
a. Infiltrative in nature
b. CT scan shows hyperdense lesion
c. Sirolimus acts by causing inflammatory process at the redundant spaces of
the malformation
d. Lined by endothelial lined wall with limited stroma

4. Tracheoesophageal fistula
a. Associated with charge
b. Shows bubble on chest radiography
c. Presents with respiratory distress at birth
d. Endoscopic is mandatory for diagnosis
e. Commonest H type

5. Nasal glioma
a. Intracranial connection with subdural csf
b. 60% intranasal
c. Enlarges with crying/straining
d. Ependymal cells seen on histology
e. Arise from lateral wall of nose in intranasal type

6. Paeds tracheostomy
a. Vertical skin incision is made in emergency
b. Hyperextended to expose thoracic trachea?
c. First change of trachy tube by parents
d. Stay suture
OTOLOGY

NOV 2007
1. Meniere’s disease
a. Associated with constipation
b. Fluctuation hearing loss
c. Mixed hearing loss
d. Potent vestibular sedative is best treatment
e. Endolymphatic surgical operation is treatment of choice
TTFTF

2. Inner hair cells


a. Are cylindrical in shape
b. Have more contact with the cochlear nerve than the outer hair
cells
c. Outer hair cells with efferent and afferent supply
d. Are more attached to tentorium than the outer hair cells
FFTF

3. Caloric test
a. We need to move the pt head 30 degree to make posterior
semicircular channel at horizontal position
b. Is by run the fluid at 30 degree for 40 s
c. With cold water, nystagmus to ipsilateral side
d. Nystagmus detected after finidhing water injection immediately
FTFF

4. Chondroma
a. Can lead to deafness
b. Common in children
TF

5. Otitic barotraumas
a. Occurs during airplane descend
b. Occurs when the different in the atmosphere pressure 80-90mm
H2O
c. More common when there is nasal block
d. Grommet is needed
TTT
MAY 2008
1. PTA graph
a. Cochlear pathology ….
b. Stapedial reflex decay at 1 KHz is at 60 dB
c. Recruitment – lesion of cochlear, poor candidate for hearing aids
d. Auditory neuropathy cannot be detected by OAE
??TT

2. Regarding embryology of inner ear


a. Stapes is the first bone to ossify
b. Malleus is from the dorsal end of Meckel’s cartilage
c. Ossification of the ossicles is between 16w and 24w
d. EAC is completely mature at puberty
FTTT
3. Sudden SNHL occurs in the following
a. PAN
b. Bahchets
c. Wegeners’ granulomatosis
d. Relapsing polychondritis
e. Otosclerosis
TTTTF
4. Endolymphatic hydrops
a. Syphilis
b. Sx in Meniere is relieved by the diuretic
c. Paget’s disease
d. Multiple sclerosis
e. BPPV
TTTFF
5. In Meniere’s disease
a. Vestibular neurectomy is better than succus decompression
b. Ecog confirms the diagnosis of Meniere’s
c. Potent vestibular sedative is best treatment
d. Associated with constipation
FTTT
6. Acoustic neuroma
a. Is a commonest CP angle tumour
b. Is commonly arises from inferior vestibular nerve
c. If tumour is very large but serviceable hearing, MCF approach is
done
d. Severe imbalance is a symptom
e. Lesion 2mm can be imaged quite clearly using gadolinium ( Gd
DPTA)
TFFFT
7. Carotid body tumour
a. Is familial
b. Is common in female
c. Derived from neural crest cell
d. Rarely metastasize
e. Poorly encapsulated & extremely vascular like hemangioma
TFTTT
8. Cholesterol granuloma
a. Cholesterol crystal surround by giant cell
b. Occur independently without cholesteatoma
c. Develop from extravasated blood clots
d. Blue ear drum
e. Occurs secondary to otitis media with effusion

TTTTT
9. Cochlear implant
a. Labyrinthitis ossificans is a relative contraindication
b. Intraoperative neural ?.. is use for mapping
c. Cochleostomy is done in scala vestibule
d. Straight electrode is done in recurrent case
e. Bilateral SNHL
TTFTT
10. Cholesteatoma
a. Primary cholesteatoma occur in middle ear infection
b. Secondary acquired cholesteatoma occur in migration through
perforated TM
c. They should has a matrix squamous epithelium
d. Epidermoid cholesteatoma is a histological variant
e. In congenital cholesteatoma arises from embryonic epidermal
cell

FTTFT
11. Otosclerosis
a. Commonest side posterior part of oval window
b. Commonly involved otic capsule
c. Autosomal recessive
d. Patient unable to hear in noisy environment
e. Blue mantle of mannase seen through naked eye
f. Usually bilateral
F T F F FT
12. Temporal bone fracture
a. Longitudinal fracture rarely spare the ossicle
b. Blood is coming out in EAC due to fracture of the roof of
tympanic cavity
c. Facial nerve involvement is common in longitudinal fracture
d. The weakest point is between the squamous-petrous junction
FTFF

13. Fracture of petrous bone


a. Lacrimation is absence if involved tympanic segment
b. Commonest side iatrogenic is at 2nd genu
FT

14. FN paralysis
a. In Stapedial arch , the contralateral reflex is affected by central
pathology
b. EnocG is used in bilateral Facial nerve paralysis
TF
15. Facial recess
a. The facial recess is lateral to sinus tympani
b. Chorda tympani is in its wall
c. Tympanic sulcus is medially
d. Is approach by posterior tympanotomy
TTFT
16. Causes of persistent discharge mastoid cavity
a. Adequate meatoplasty
b. Large mastoid bowl
c. Ascending infection from Eustachian tube
d. Cholesteatoma in sinodural angle
e. Granulation tissue in ME
FTTTT
17. Speech audiometry
a. SRT
b. Is a temporal discrimination
c. In children free field test is presented as dB HL
TF

18. Hearing loss


a. Normal individu should have 0 Db at all frequency
b. Loudness is increase 20 times when the intensity is increase
20Db
c. Whisper at 1m is at 30Db
FFF

19. Herpes zoster oticus


a. Is associated with diffuse vesicular eruption in EAC
b. Viral inflammation in geniculate ganglion
c. Facial nerve palsy is due to swelling in the geniculate ganglion
d. Geniculate ganglion is easily accessible for surgical decompression

TTTF

20. Impedance audiometry


a. For neonate we used 226Hz
b. Tympanometry measurement of acoustic admittance with
sound pressure
FT

21. ERA
a. ABR
b. CERA , done during sleep

?F
NOV 2008

1. Regarding glomus tumour


a. It is commonest tumour within the temporal bone
b. Glomus tympanicum originates from jugular bulb
c. Its stains with chromium
d. It is a vascular tumour
e. It is fast growing
TFFTF

2. Regarding Meniere’s disease


a. It is due to an increase in volume of perilymphatic space
b. It has premonitory symptom of fullness in the ear
c. There is fluctuation in the hearing level
d. There is a fluctuating tinnitus
e. Betahistine hydrochloride is the treatment
FTTTT

3. Aminoglycoside that has more vestibulotoxic than cochleotoxic


properties includes
a. Streptomycin
b. Gentamycin
c. Kanamycin
d. Tobramycin
e. Neomycin
TTF?F

4. Regarding temporal bone fracture


a. Incudo-malleolar joint is the commonest site if traumatic
separation
b. The longitudinal fracture lines continues across the roof of the
middle ear space
c. In longitudinal fracture, the site of facial nerve injury is proximal
to the geniculate ganglion
d. Facial nerve paralysis is commoner in transverse fractures
e. Transverse fracture is commoner than longitudinal fracture
FTFTF

5. Regarding nystagmus
a. Vertical nystagmus is seen in peripheral lesion
b. Failure of fixation to suppress nystagmus is a sign of central
pathology
c. Positive Dix- Hallpike tests showed immediate onset nystagmus
d. The central type disappears on repetition
e. Vetibular nystagmus is exaggerated when the eye looks in the
direction of the slow component
FTFFF

6. Regarding patulous eustachian tube


a. Autophony is the presenting symptom
b. Patients with mild symptoms require only reassurance
c. The tympanic membrane moves with respiration
d. Systemic decongestant helps to relieve symptoms
e. Insertion of a ventilating tube is one of the treatment options
TTTFT
7. Regarding orbital apex syndrome
a. In a young adult, it is a complication of anterior ethmoidal
sinusitis
b. There is marked bilateral proptosis
c. There is a slow onset of blindness on the affected side
d. Patient present with opthalmoplegia
e. Red color vision impairment necessitates early surgical
intervention
FFFTT

8. Regarding surgery for otosclerosis


a. The risk of total hearing loss in operated ear is less than 1%
b. Stapes mobilization is effective for short term period
c. Maximum speech discrimination score should be less than 70%
d. Postoperatively fluctuating hearing loss indicate dislocation of
the prosthesis
e. Persistent vertiginous symptoms immediately after surgery
indicate labyrinthitis
TTFFT

9. The following statement are true


a. A neonate is able to discriminate intensity and frequency of
sound
b. By 2 months the child is able to differentiate phonemes
c. The frequency of humming voice is between 250 – 1000 hz
d. The frequency of ‘sss’ voce is between 3000-8000 Hz
e. The Manchester rattle produces sound which has a frequency
between 2500-3500 Hz
?TTTF
10. Endolymphatic hydrops
a. Occurs in hypothyroidism
b. In Meniere disease is characterized by low frequency
sensorineural hearing loss
c. Demonstrate a flamingo blush on the promontory on otoscopy
d. Is characterized by enhancement of the summating potential on
electrocochleography
e. Is associated with canal paresis on caloric testing

TTFTT
11. The following are the differential diagnosis of a cerebellopontine angle
mass
a. Vestibular schwannoma
b. Meningioma
c. Cholesteatoma
d. Aneurysms
e. Glomus tympanicum
TTTTF
12. The following drugs causes ototoxicity
a. Cisplatin
b. Frusemide
c. Aspirin
d. Ethambutol
e. Rifampicin
TTTFF
13. The following are true of benign paroxysmal positional vertigo
a. It is the commonest cause of peripheral vertigo
b. It is predominant due to calcium carbonate deposition on the
cupula of the lateral semi-circular canal
c. A predisposing factor for BPPV is head injury
d. The direction of nystagmus does not change on repeated testing
e. Labyrinthine sedative is the treatment of choice
TFTTF

14. In sensorineural hearing loss


a. Sudden onset is frequently unilateral
b. Noise induce is due to loss of auditory neurons
c. Salicylate deafness is reversible
d. Perilymph fistula presents as fluctuating hearing loss
e. With vertigo indicate a poor prognosis
TFTTT

15. Regarding speech audiometry


a. It depends on temporal discrimination
b. Speech discrimination test is done with open-sets of test
material
c. Speech identification is measured in speech perception test
d. A score of 0% suggest a patient not understanding
e. Half optimum speech recognition threshold level correlates well
with pure tone threshold over 500-2000 Hz
TFTTT

16. Regarding impedence audiometry


a. Acoustic reflex attenuates signal/noise ratio for speech
b. Tympanometry is a measure of acoustic admittance of ear canal
as a function of pressure
c. Contralateral acoustic reflex test is sensitive to central
pathologies
d. Alcohol elevates the acoustic threshold reflex
e. 226 Hz tympanometry is suitable for neonate

FTTTF
17. The following theories are correct regarding presbycusis
a. Neural presbycusis refers to degeneration and loss of hair cells
b. Sensory presbycusis refers to degeneration and loss of neural
elements
c. Metabolic presbycusis refers to biochemical abnormalities in the
inner ear
d. Mechanical presbycusis refers to degeneration of inner ear
supportive elements
e. Mixed presbycusis refers to degeneration of the hair cells ad
neural elements
FFTTT
May 2012

1. Temporal lobe abscess


a. ( ) Nominal aphasia
b. ( ) Homonymous heminopia
c. ( ) Auditory hallucination
d. ( ) Nystagmus
e. ( ) Intentional tremor

2. Carotid body tumour


a. ( ) Aka non chromaffin paraganglioma
b. ( ) Arises from carotid body chemoreceptor
c. ( ) Classified by Chandler
d. ( ) Hypoglossal nerve at risk in surgery

4. Better outcome of CI in children is determine by


a. ( ) Sudden onset SNHL
b. ( ) Older child
c. ( ) Longer duration of Ci wear
d. ( ) Aural rehabilitation
MAY 2013

1. Facial nerve palsy


a. ( ) Schwannoma is the commonest tumour of facial nerve
b. ( ) 1st genu injured commonly during otology surgery
c. ( ) In ASOM is caused by compression
d. ( ) Full recovery in 40% of Bell’s palsy

2. Traumatic facial nerve injury


a. ( ) Stapedial reflex lost in injury at the tympanic segment
b. ( ) Schirmer’s test negative in labyrinthine segment injury
c. ( ) Electrogustometry is a useful test
d. ( ) ENoG shows normal recording on the second day
e. ( ) Incomplete eye closure is seen in HB Grade III facial palsy

3. Congenital facial palsy seen in


a. ( ) Mobius syndrome
b. ( ) Melkersson Rosenthal syndrome
c. ( ) Hemi facial microsomia
d. ( ) Lyme disease
e. ( ) Ramsay Hunt syndrome

4. Typical audiometric findings in Meniere’s disease


a. ( ) AP/ SP ratio more than 3
b. ( ) Cochlear microphonic is normal
c. ( ) Duration of SP is long
d. ( ) Sp corresponds with negative potential
e. ( )?amplitude…

5. Ototoxicity in perforated TM by topical drugs


a. ( ) Povidone
b. ( ) Chlorhexidine
c. ( ) Ofloxacin
d. ( ) Gentamycin
e. ( ) Alcohol

6. Ototoxicity by systemic drugs


a. ( ) Kanamycin
b. ( ) Ethacrynic acid
c. ( ) Erythromycin
d. ( ) Furosemide
e. ( ) Salicylate

7. Causes of subjective tinnitus


a. ( ) Presbycusis
b. ( ) Anxiety
c. ( ) Barotrauma
d. ( ) Aspirin
e. ( ) Multiple sclerosis
f. ( ) NSAIDs
8. BERA
a. ( ) Interneuron wave 5 latency indicate AN (acoustic neuroma)
b. ( ) Wave 2 indicates superior olivary nucleus
c. ( ) It measure the impulse in 1 st20ms
d. ( ) Wave I enhanced by electrocochleography

9. Nystagmus
a. ( ) Congenital nystagmus is pendular type
b. ( ) Positional nystagmus is rotatory
c. ( ) All peripheral nystagmus is horizontal and biphasic
d. ( ) Fast component is peripheral compensation
e. ( ) Change in direction is typical of peripheral nystagmus

10. Videonystagmography
a. ( ) Supra ocular lesion
b. ( ) Can diagnose cerebellar lesion
c. ( ) For vergence
d. ( ) Can assess supranuclear
e. ( ) Can differentiate compensated and non compensated
nystagmus

11. Otitis media with effusion


a. ( ) Due to inflammation at ET epithelium
b. ( ) Has bimodal peak, 1st is in 2 years
c. ( ) Tympanogram is gold standard
d. ( ) Can resolve spontaneously

12. OME
a. ( ) Treated with topical ear drop
b. ( ) Antihistamine useful in paed
c. ( ) Grommet can minimize recurrence
d. ( ) Adenoidectomy before 2yo help
e. ( ) Pneumococcal vaccine reduce incident

13. Swimmers ear


a. ( ) Pulling pinna up causes pain
b. ( ) Caused by hair follicle infection
c. ( ) Caused by staph aureus
d. ( ) Treatment by topical antibiotics
e. ( ) Granulation tissue seen on the floor

14. Optokinetic
a. ( ) Rapid movement in vertical direction
b. ( ) Never fatigue
c. ( ) Allow someone to see fast moving object
d. ( ) Is specific to detect vestibular disorder
15. NIHL
a. ( ) Threshold for temporary threshold shift is at 100dB
b. ( ) In temporary threshold shift, the stereocilia of OHC is
destroyed
c. ( ) Affect frequency half an octave higher than the frequency
of the noise
d. ( ) Affect basal turn commonly
e. ( ) 3kHz dip can be seen in PTA
f. ( ) TTS an protect against a sudden noise in 140dB

16. Caloric test


a. ( ) In Meniere’s shows unilateral deficit
b. ( ) AN is unilateral deficit
c. ( ) Recurrent vestibulopathy is unilateral
d. ( )?? unilateral not affected by age
e. ( ) Central vestibular disorder gives unilateral deficit

17. CSOM complication


a. ( ) Extradural abscess is the commonest
b. ( ) Contrast enhanced ct done to look for meningeal infection

18. OAEs
a. ( ) Affected by body movement
b. ( ) Detect threshold
c. ( ) Portable OAE is an objective test
d. ( ) DPOAE is frequency specific
e. ( ) Present in sensorineural hearing loss
f. ( ) Useful in identification of deafness

19. Malignant otitis externa


a. ( ) Represent skull base osteomyelitis
b. ( ) Radiation increase risk
c. ( ) Caused by staphylococcus aureus
d. ( ) Surgical debridement is the mainstay of treatment
e. ( ) Tech 99 monitor disease progress
f. ( ) Otorrhea is the main presenting symptom

20. Ototoxicity
a. ( ) Aminoglycoside is cochleotoxic

21. Hearing testing


a. ( ) Distraction test for 1 yo child
b. ( ) Play audiometry to detect threshold

23. Regarding clinical test in vestibular disorder


a. ( ) Cerebellar testing is positive in congenital nystagmus
b. ( ) Caloric test, test the LSCC
c. ( ) Posturography is to determine posterior column pathology
d. ( ) Electrocochleography is positive in acoustic neuroma
e. ( ) Romberg test is positive in Meniere’s disease
MAY 2014

1. Which are the hearing preservation approach


a. ( ) Retrolabyrinthine
b. ( ) Retrosigmoid
c. ( ) Translabyrinthine
d. ( ) Transcochlear
e. ( ) Middle fossa approach

2. Differentials of CPA mass


a. ( ) Cholesteatoma
b. ( ) Aneurysm
c. ( ) Schwannoma
d. ( ) Glomus tympanicum
e. ( ) Meningioma

3. In cervico-aural fistula
a. ( ) The upper fistula can open beyond the cartilaginous of EAC
b. ( ) Type 2 opening is below angle of mandible
c. ( ) During surgery , facial nerve is at risk
d. ( ) Comprise of 15% of all branchial anomalies
e. ( ) Type 1 is from mesodermal origin

4. Stapedial reflex
a. ( ) Can differentiate sensory or neural hearing loss
b. ( ) Can be determine auditory neuropathy
c. ( ) Can differentiate cochlear (sensory) vs retrocochlear
(neural) lesion
d. ( ) Can detect brainstem lesion
e. ( ) Its to detect middle ear problem

5. Peripheral nystagmus
a. ( ) Is uniphasic
b. ( ) In lateral SCC BPPV is rotatory nystagmus during Dix –
Hallpike manoeuvre
c. ( ) Worsened by optic fixation
d. ( ) Frenzel glass can eliminate the optic fixation
e. ( ) Associated with nausea, vomiting
f. ( ) Follow alexander law

6. Temporal bone fracture


a. ( ) Ear bleed suggestive of fracture of roof of middle ear
b. ( ) Middle ear structure are rarely escaped from longitudinal
type fracture
c. ( ) Prognosis of SNHL in transverse fracture is good
d. ( ) Fusion area between squamous part and petrous part of
temporal bone is a weak point
e. ( ) Facial nerve injury usually related to llongitudinal fracture

7. Corticol mastoidectomy is indicated in


a. ( ) Bone anchored hearing aid implantation
b. ( ) Cochlear implantation
c. ( ) Middle ear implant
d. ( ) Stapedectomy
e. ( ) Saccular decompression

8. Regarding meniere disease


a. ( ) Due to fluid overload in endolymph
b. ( ) Meniere disease is treated with high dose betahistine
c. ( ) Confirm by electrocochleagraphy
9. Cause of fluctuating hearing loss
a. ( ) Meniere’s disease
b. ( ) Viral labyrinthitis
c. ( ) Perilymphatic fistula
d. ( ) Ossicular chain disruption
e. ( ) Otosclerosis

9. Relative contraindication of cochear implant


a. ( ) Labyrinthine ossificans
b. ( ) Incomplete partition of cochlear
c. ( ) Cochlear nerve hypoplasia
d. ( ) Michel deformity
e. ( ) Middle ear effusion

10. Prognosis factor of cochlear implant


a. ( ) Onset of hearing loss
b. ( ) Duration of hearing loss
c. ( ) Language
d. ( ) Insertion of electrode
e. ………………………………..

11. Anatomical relationship


a. ( ) Oval window = utricule
b. ( ) Cochlear aquaduct = scala tympani
c. ( ) Scala tympani = scala vestibule
d. ( ) Round window = scala vestibule
e. ( ) SCC to saccule

12. Regarding OAE


a. ( ) OAE can be used to estimate CHL
b. ( ) Cannot detect HL less than 30db
c. ( ) Need stimulus
d. ( ) Can differentiate neural lesion

13. Regarding hearing aid


a. ( ) BAHA limited in 60 dB loss
b. ( ) Analog hearing aid is effective in overcoming recruitment in
SNHL
c. ( ) Peak clipping in useful in prolonging battery life

14. Regarding otosclerosis are true


a. ( ) Cochlear implant is indicated in severe HL of otosclerosis
patient
b. ( ) Operation is contraindicated in the presence of scwartz sign

15. Definition in audiology


a. ( ) dB SL is hearing above the normal statistical population
b. ( ) dB HL is in comparison with sound pressure level
c. ( ) dB SPL hearing in comparison with normal threshold of the
person
d. ( ) sound pressure level taken as reference in ? dB SL is 20
micropascal
MAY 2015

1. Non organic hearing loss


a. ( ) BSER is gold standard
b. ( ) Stenger’s test can be performed bedside
c. ( ) ECochG is essential to detect threshold
d. ( ) Serial audiometry is inconsistent
e. ( ) Stapedial reflex seldom of value

2. Caloric test is contraindicated in


a. ( ) Epilepsy
b. ( ) Migraine
c. ( ) Acoustic Neuroma
d. ( ) Comatose patient
e. ( ) Psychotic patient

3. Hearing aids
a. ( ) BAHA for severe bilateral SNHL
b. ( ) BAHA for canal atresia
c. ( ) Mode of compression to improve in digital hearings aids
d. ( ) Ear mould improve hearing in low frequency

4. MOE
a. ( ) Skull base may involve
b. ( ) Caused by pseudomonas aeruginosa
c. ( ) Treated by gentamicin
d. ( ) May present like temporal bone carcinoma

5. Cochlear implant
a. ( ) Can in 2 years old
b. ( ) 2 months can wear hearing aids
c. ( ) 2 years can refer speech therapy
d. ( ) BAHA?

6. Acoustic reflex
a. ( ) Reflex is absent in middle ear pathology
b. ( ) Reflex threshold is 55dB above normal threshold
c. ( ) Normal contralateral reflex threshold implies intact
superior olivary complex
d. ( ) Provides parameters for monitoring of facial nerve function
e. ( ) Loud sound directed into either ear will cause bilateral
contraction

7. Tuning fork test


a. ( ) Rinne is positive when BC > AC
b. ( ) Weber is centralized in bilateral dull TM
c. ( ) Rinne is negative when CHL > 5dB
d. ( ) Weber is lateralized to disease ear with underlying dead ear
e. ( ) Auditory neuropathy can be detected

8. Acoustic neuroma
a. ( ) Brainstem implant is indicated after bilateral acoustic
neuroma

9. Hearing
a. ( ) 60dB is normal hearing in conversation
b. ( ) Using gun, hearing loss in both ear will be affected similarly
10. Fully developed structure at birth
a. ( ) Malleus
b. ( ) Inner ear
c. ( ) Mastoid
d. ( ) TMJ
e. ( ) Shenoid sinus

11. Meniere’s disease


a. ( ) Associated with inflamed TM
b. ( ) Vestibular hydrops associated with fluctuating SNHL
c. ( ) Tumarkin drop is one of the variant
d. ( ) Vertigo is episodic lasting more than 24 hours
e. ( ) Contralateral ear involved in within 6 month

12. Acoustic neuroma


a. ( ) Surgery if tumour less than 2cm
b. ( ) Translabyrinthine approach if tumour > 4cm
c. ( ) Middle cranial fossa approach treatment of choice in
preserve hearing
d. ( ) 65% of CPA tumour
e. ( ) Present with vertigo
f. ( ) Facial numbness indicate extent of tumour to Meckel’s
cave

13. Intermittent vertigo


a. ( ) Acoustic neuroma
b. ( ) Perilymphatic fistula
c. ( ) Meniere’s disease
d. ( ) Multiple sclerosis
e. ( ) Otosclerosis

14. Exostosis
a. ( ) Bilateral
b. ( ) Cancellous bone
c. ( ) Floor only
d. ( ) No treatment if asymptomatic
15. Regarding ear pinna
a. ( ) Pain in allergic dermatitis
b. ( ) Peu de orange in lymphoedema
c. ( ) Chondritis cause by trauma
d. ( ) Erysipelas can cause systemic sepsis
e. ( ) Cellulitis cause by gram negative

16. Temporal bone fracture


a. ( ) Transverse due to occiput trauma
b. ( ) Middle ear fracture usually at incudostapedial
c. ( ) CSF leak from posterior cranial fossa
d. ( ) CTscan with contrast is to detect fracture line
17. SNHL
a. ( ) Syphilis
b. ( ) Vertebra basilar insufficiency
c. ( ) Lateral medullary syndrome
d. ( ) Cochlear otosclerosis
e. ( ) Meniere’s disease

18. Facial nerve identification


a. ( ) Medial to digastric ridge
b. ( ) Lateral to tympanomastoid suture
c. ( ) Superior to oval window
d. ( ) Inferior to posterior SCC
e. ( ) Anterior to processus cochleariformis
19. Ossiculoplasty
a. ( ) Belfast rule of thumb
b. ( ) Achievement less than 30 AB gap
c. ( ) Sudden reduce in hearing may cause by prosthesis
displacement

20. Vestibular neuritis


a. ( ) Vertigo lasting more than 24 hours
b. ( ) With hearing loss
c. ( ) Nystagmus to contralateral
d. ( ) ENG shows reduce response
e. ( ) Labyrinthectomy is treatment of choice
NOV 2015

1. Vestibular rehabilitation technique


a. ( ) Habituation
b. ( ) Taichi
c. ( ) Compensation
d. ( ) Posturography to measure outcome
e. ..

2. Causes of Meniere’s disease


a. ( ) Anaemia
b. ( ) Hypertension
c. ( ) Hyperthyroid
d. ( ) DM
e. ( ) Hyperlipidaemia

3. Perichondritis
a. ( ) Commonly present peau de orange
b. ( ) Erysipelas due to skin abrasion
c. ( ) Sebarrhoic dermatitis due to strep group A
d. ( ) Allergy to drug at dependent area
e. ( ) Cellulitis can present as systemic toxicity

4. Canal cholesteatoma
a. ( ) Old age
b. ( ) Abnormal epithelial migration
c. ( ) Treatment is surgery
d. ( ) Ballooning canal

5. SNHL can be seen in


a. ( ) Meniere’s disease
b. ( ) Lateral medullary syndrome
c. ( ) Syphilis
d. ( ) Cochlear otosclerosis
e. ( ) Vertebra-basilar insufficiency / PICA

6. Temporal bone surgery


a. ( ) In modified radical mastoidectomy, ossicles preserve
b. ( ) Endaural incision for attic disease
c. ( ) Radical mastoid is removing all ossicles
d. ( ) Radical mastoid ET is blocked
e. ( ) Endaural incision better in visualizing antrum

7. Clinical masking
a. ( ) AC same ear more than 20 dB
b. ( ) AB gap more than 20 dB should be done
c. ( ) Written in dB SPL
d. ( ) Using white noise
8. Universal hearing screening
a. ( ) Done in all baby
b. ( ) OAE is frequency specific
c. ( ) ABR min 20 dB HL
d. ( ) Mild hearing loss usually missed
e. ( ) Repeat after 3 months if fail

9. Acoustic reflex
a. ( ) First detected is the threshold
b. ( ) Can present in contralateral ear
c. ( ) Minimal threshold is 60 dB
d. ( ) Highest frequency is 2kHz
e. ( ) The afferent nerve that involve in non-acoustic reflex
pathway is trigeminal nerve

10. Cochlear implant


a. ( ) As early as possible in prelingual
b. ( ) Hearing aid is pre-requirement
c. ( ) Require mapping pre- op

11. Electroneurography
a. ( ) Measure function of neuron
b. ( ) No benefit in bilateral facial nerve palsy
c. ( ) Need supramaximal stimulation
d. ( ) Day 1 effective

12. Exostosis
a. ( ) Deep part of canal
b. ( ) Bilateral
c. ( ) Can cause cholesteatoma
d. ( ) Cause by swimming in cold water
e. ( ) Cortical mastoid is treatment

13. Corticol mastoidectomy


a. ( ) For chronic mastoiditis
b. ( ) Done in chronic otitis media
c. ( ) Treatment for malignant otitis externa
d. ( ) Meatoplasty usually done
e. ( ) Done in endolymphatic duct decompression

14. Cholesteatoma
a. ( ) Has cholesterol in feature
b. ( ) The capsule is lined with stratified squamous epithelium
c. ( ) Head of malleus is usually eroded
d. ( ) Automastoidectomy will be symptom free
e. ( ) MRI hyperattenuation in T1 & enhanced with gadolinium

15. Surgery for CSOM


a. ( ) Endaural approach leads to access of antrum
b. ( ) Radical mastoidectomy lead to low incidence of discharge
c. ( ) Cholesteatoma matrix over semi-circular canal erosion is
not usually removed
d. ( ) SNHL is result of stapes manipulation
e. ( ) Post op haemorrhage is seen after hypotensive anaesthesia
16. Cortical mastoidectomy is indicated in
a. ( ) Chronic mastoiditis
b. ( ) Keratosis obsturan
c. ( ) CSOM
d. ( ) Endolymphatic sac decompression
e. ( ) Malignant otitis externa

17. Major criteria for MOE


a. ( ) Positive culture for pseudomonas
b. ( ) Diabetis mellitus
c. ( ) Failed medical treatment
d. ( ) Old age
e. ( ) Presence of microabscess

18. Regarding the inflammatory disorder of auricle


a. ( ) Erysipelas begins with an abrasion of the epidermis
b. ( ) Allergy to ear drop demonstrate dermatitis in its dependent
part
c. ( ) Cellulitis is characterised by systemic toxicity
d. ( ) Perichondritis typically demonstrate area of peau de orange
e. ( ) Seborrheic dermatitis is caused by group A streptococcus

19. Regarding the cochlea


a. ( ) Organ of corti converts sound stimulus to electrical signals
b. ( ) Labyrinthine artery is the only artery blood supply
c. ( ) The modiolus contains spiral ganglion and cochlear nerve
d. ( ) The outer hair cell are more resistant to ototoxic drugs
e. ( ) Inner hair cell more important in hearing

20. Posterior tympanotomy allow surgeon to


a. ( ) Inspect the epitympanum ? exposure of mesotympanum
b. ( ) Perform cochleostomy
c. ( ) Plug the eustachian tube
d. ( ) Perform stapedectomy
e. ( ) Decompress tympanic segment of facial nerve

21. Regarding principle of mastoid surgery


a. ( ) Radical mastoidectomy involves removal of all ossicles
b. ( ) Modified radical mastoidectomy disease ossicle is removed
c. ( ) Atticotomy is best approach via endaural incision
d. ( ) Corticol mastoidectomy requires a meatoplasty
e. ( ) Post op hearing is better in canal wall up compared to canal
wall down
MAY 2016
1. Siegel speculum
a. ( ) Magnify small TM perforation
b. ( ) Insufflate power into external ear and middle ear
c. ( ) Fistula test
d. ( ) Apply negative pressure
e. ( ) Assess mobility of TM

2. Clinical manifestation of intra-temporal complication of CSOM include


a. ( ) FN palsy
b. ( ) Peripheral Nystagmus
c. ( ) Nominal aphasia
d. ( ) Fluctuating sensorineural HL
e. ( ) Post auricular swelling

3. BPPV
a. ( ) Due to ototoxicity
b. ( ) Gentamicin is treatment
c. ( ) More in elderly
d. ( ) Common after viral fever

4. Regarding glomus tumour of the temporal bone


a. ( ) It is called chromaffin paragangliomas
b. ( ) Glomus tympanicum arises from the middle ear cleft
c. ( ) Tumour confined to middle ear is best treated by lateral
temporal bone resection
d. ( ) Hypoglossal nerve palsy is a sign of glomus jugulare
e. ( ) Radiotherapy causes a high percentage of malignant change

5. Tulio phenomenon
a. ( ) Meniere’s disease
b. ( ) Otosclerosis
c. ( ) BPPV
d. ( ) Labyrinthitis
e. ( ) Congenital syphilis

6. Differential diagnosis of CPA


a. ( ) Glomus jugulare
b. ( ) Secondary brain tumour (metastases)
c. ( ) Epidermoid cyst
d. ( ) Cholesterol granuloma
e. ( ) Berry aneurysm

7. Regarding Otosclerosis
a. ( ) Is a disease limited to otic capsule
b. ( ) Surgery is contraindicated for the only hearing ear
c. ( ) Is an autosomal recessive condition
d. ( ) Commonly at the posterior part of oval window
e. ( ) It is equally common in male and female

8. PTA
a. ( ) Start at 500hz
b. ( ) Above 7 yo
c. ( ) Inter octave testing is 20dB
d. ( ) NIHL dip at 3kHz
NOV 2016

1. The following are true regarding glomus tumour of temporal bone


a. ( ) It is called chromaffin paraganglioma
b. ( ) Tumour arise from middle ear cleft is called glomus
tympanicum
c. ( ) Tumour confined to middle ear cleft best treated with
lateral TBR
d. ( ) Hypoglossal nerve is a sign of glomus jugulare
e. ( ) Radiotherapy causes a high percentage of malignant change

2. Regarding cochlear implant


a. ( ) It is indicated as early as 1 year old
b. ( ) Multichannel send different electrical signals to different
sites in the cochlea
c. ( ) Patient with no residual hearing should not have the
implant
d. ( ) Pre-evaluation counselling play a minimal role in suitable
candidates for implant
e. ( ) Meningitis is unusual complication
f. ( ) Aural rehabilitation include leaning sign language

3. The following are true of hearing test


a. ( ) When performing masking, the non tested ear is exposed to
pure tone sound
b. ( ) Pathology of the cochlear show a normal brainstem evoke
response
c. ( ) Otoacoustic emission measures hearing threshold level
d. ( ) Distraction test of hearing is performed in a child of 5 years
e. ( ) Visual reinforcement audiometry is used for testing a child
of 3 years

4. Otosclerosis
a. ( ) Characterised by Otodystrophy of the otic capsule
b. ( ) Equally common in male and female
c. ( ) It is common in children
d. ( ) Hearing aid is treatment option
e. ( ) It is categorised into fenestral and retrofenestral

5. Regarding the inflammatory disorder of the auricle


a. ( ) If allergic to eardrops demonstrate dermatitis in its
dependant part
b. ( ) Perichondritis typically demonstrate area ‘Peau de orange’
c. ( ) Seborrheic dermatitis is caused by group-A streptococcus
d. ( ) Cellulitis is marked by high grade fever
e. ( ) Erysipelas begins with and abrasion of the epidermis

6. Regarding cholesteatoma
a. ( ) Has matrix lined by stratified squamous epithelium
b. ( ) Contain cholesterol
c. ( ) Demonstrate intense enhancement on MRI with gadolinium
d. ( ) It is symptom free in automastoidectomy
e. ( ) The head of malleus is the most susceptible ossicles to
erosion
7. Findings in Meniere’s disease
a. ( ) Endolymphatic hydrops is visualized on MRI with
gadolinium
b. ( ) Frequency o attacks are more often in early phase of
disease
c. ( ) The presence of tone decay
d. ( ) The presence of over recruitment
e. ( ) The rate of bilateral disease at initial stage is 50%

8. Vestibular neuritis
a. ( ) Is a reactivation of latent infection of Herpes Simplex type I
b. ( ) Presents with rotatory vertigo
c. ( ) Shows and impaired vestibular ocular reflex to the normal
side
d. ( ) Is treated with methylprednisolone
e. ( ) Has a low recurrence rate

9. Regarding the procedure for vestibular disorder


a. ( ) The presence of nystagmus after head shake test,
suggestive of a central vestibular pathway lesion
b. ( ) The head trust test (Halmagyi) differentiates between
vestibular and brainstem lesion
c. ( ) Snellen chart is used in the test for oscillopsia
d. ( ) Dix-Hallpike tests is used to confirm posterior semi-circular
canal involvement
e. ( ) Rotatory chair test evaluates saccule

10. Regarding acute suppurative otitis media


a. ( ) It is more common in children
b. ( ) The incidence is lower in breastfed children
c. ( ) The first stage of infection is characterized by bulging of the
tympanic membrane
d. ( ) It has a tendency towards spontaneous healing
e. ( ) The most common pathogen is group A streptococcus

11. Regarding chronic suppurative otitis media


a. ( ) Endaural approach gives acess to the antrum
b. ( ) Radical mastoidectomy has low incidence of long term
discharge
c. ( ) A cholesteatoma matrix is left over large semi-circular canal
fistula
d. ( ) Sensorineural hearing loss is caused by excessive
manipulation of the stapes footplate
e. ( ) Postoperative haemorrhage is a risk after hypotensive
anaesthesia
12. Regarding caloric test
a. ( ) It evaluates the response of the posterior semi-circular
canal to different temperature
b. ( ) The lateral semi-circular canal is made vertical by lying the
patient 30 degree to horizon
c. ( ) Two different temperature are used 30C and 44C
d. ( ) In the presence of perforation the Kobrak’s test is used
e. ( ) Tests the reaction of the macula to different temperature

13. Regarding the choice of the hearing rehabilitation


a. ( ) Cochlear implant is performed in children as young as 1
year old
b. ( ) Hearing aids are prescribed to children as young as 1 year
old
c. ( ) Brainstem implant is indicated in cases of post operative
bilateral acoustic neuroma
d. ( ) Speech therapy helps children below 2 years
e. ( ) A child with severe bilateral sensorineural hearing loss
benefits from BAHA

14. Regarding brainstem evoked response (BERA)


a. ( ) Conductive hearing loss causes delayed wave I latency
b. ( ) Brainstem auditory dyfunction causes delayed wave V
latency
c. ( ) In infant there is a delayed wave I-V latency
d. ( ) At high click stimulus intensity of 85dB, wave V latency Is
normally about 4.5msec
e. ( ) Wave V is due to response at superior colliculus

15. Regarding otoacoustic emission ( OAE)


a. ( ) It is frequency specific in transient OAE
b. ( ) It test the function of the inner hair cells
c. ( ) It is useful in diagnosing acoustic neuroma
d. ( ) Spontaneous otoacoustic emission is present in all normal
subjects
e. ( ) It helps to diagnose a case of non organic hearing loss

16. Regarding pure tone audiometry


a. ( ) The method employed is the modified Hughson Westlake
technique
b. ( ) Bone conduction testing ranges from 500 to 8000 Hz
c. ( ) Air conduction testing is best done with and insert
earphone
d. ( ) It is an objective method of testing
e. ( ) Bone conduction testing is masked in 40dB interaural
difference

17. Exostosis
a. ( ) Bilateral ear is common
b. ( ) Multiple swelling in ear canal
c. ( ) No need excision if asymptomatic
NOV 2017
1. NOE
a. ( ) Granulation tissue at junction of cartilage and bone
b. ( ) Clindamycin is drug of choice
c. ( ) Isotope scan is better than CT scan to identify infective
process

2. CPA mass
a. ( ) Vestibular schwannoma
b. ( ) Meningioma
c. ( ) Aneurysm
d. ( ) Glomus tympanicum
e. ( ) Cholesteatoma

3. Posterior tympanotomy allows surgeon for following


a. ( ) Cochleostomy
b. ( ) Stapedotomy
c. ( ) Decompression of 1 stgenu

d. ( ) Plug ET
e. ( ) Directly access sinus tympani

4. Vestibular neuronitis
a. ( ) Presented with hearing loss
b. ( ) Vertigo lasted more than 24 hours
c. ( ) Caloric test shows canal paresis
d. ( ) Nystagmus to affected side

5. Tuning fork test


a. ( ) Stenger test is a bedside test
b. ( ) Chimani moos test is a modified weber test
c. ( ) Rinne positive means normal hearing

6. Right normal, Left SNHL


a. ( ) Weber lateralized to right
b. ( ) Weber to left it profound SNHL
c. ( ) Rinne negative if SNHL 20-40dB
d. ( ) Rinne positive if severe profound SNHL

7. Structures adjacent to FN in ME
a. ( ) Round window
b. ( ) Stapes footplate
c. ( ) Processus cochlearformis
d. ( ) ET
e. ( ) Lateral SCC

8. Canal cholesteatoma
a. ( ) Due to abnormal epithelial migration
b. ( ) Cause ballooning of canal
c. ( ) Best treated with surgery
d. ( ) usually bilateral
9. Peripheral nystagmus
a. ( ) Name after the fast phase
b. ( ) Towards lesion in acute labyrinthitis
c. ( ) Spontaneous nystagmus reduced if remove optic
fixation
d. ( ) Dix hallpike elicit horizontal nystagmus in post BPPV

10. Meniere’s disease


a. ( ) Surgical treatment by endolymphatic sac
decompression
b. ( ) Treated with vasodilator
c. ( ) Vertigo less than half an hour
d. ( ) Low frequency SNHL

11. OAE
a. ( ) Normal OAE indicate functioning organ of Corti
b. ( ) DPOAE
c. ( ) To detect retrocochlear lesion
d. ( ) Test hair cell function
e. ( ) Detect threshold

13. Speech audio


a. ( ) Test ability to hear pure tone at 40dB
b. ( ) Normal result shows some approach 100% 25dB
c. ( ) Roll over phenomenon in CHL
d. ( ) Patient with secretory OM fail to achieve score of 100%
e. ( ) Shift normal curve to right seem in CSOM

14. PTA
a. ( ) Masking perform when there is 40dB difference AC
b. ( ) BC testing range 500-8kHz
c. ( ) AC testing always done with insert earphone
d. ( ) Objective method testing
e. ( ) All BC hearing threshold should mask
NOV 2018
1. Regarding facial nerve palsy
a. ( ) Bell palsy is the commonest cause.
b. ( ) Complete recovery is likely achieved in neuronotmesis.
c. ( ) Electroneuronography is tested within three days of
paralysis.
d. ( ) Melkersson – Rosenthal syndrome is characterized by
recurrent facial nerve palsy.
e. ( ) Topognostic test is used to determine the site of facial
nerve palsy.

2. Regarding cholesterol granuloma


a. ( ) High Resolution Computed Tomography scan differentiates
between cholesterol granuloma and cholesteatoma.
b. ( ) It is a histologic diagnosis.
c. ( ) It presents as a cystic lesion on Magnetic Resonance
Imaging.
d. ( ) The commonest site is petrous apex.
e. ( ) Total surgical excision is required.

3. Regarding ear canal disease


a. ( ) Canal cholesteatoma causes localized erosion of ear canal.
b. ( ) Exostosis is a true neoplasm.
c. ( ) Exostosis is removed with curette.
d. ( ) Keratosis obturans causes bony remodelling.
e. ( ) Osteoma is devoid of blood vessels.

4. Regarding herpes zoster oticus


a. ( ) Antiviral is indicated in prodromal phase only.
b. ( ) Caused by varicella virus.
c. ( ) Causes persistent facial nerve weakness in 90% of patients.
d. ( ) Is part of Ramsay-Hunt syndrome.
e. ( ) More common in immunocompetent patients.

5. Regarding CSF otorrhoea


a. ( ) β- trace protein is a gold standard diagnostic test.
b. ( ) CSF leak in longitudinal type of temporal bone fracture is
self–limiting.
c. ( ) Middle fossa approach is indicated in patients with normal
hearing.
d. ( ) Occurs more in transverse type of temporal bone fracture
than longitudinal type of temporal bone fracture.
e. ( ) Posterior fossa defect causes CSF leakage through
Trautmann triangle.

6. Acute otitis media,


a. ( ) causes complete facial nerve paralysis.
b. ( ) common in elderly.
c. ( ) hearing loss is the main presenting symptom.
d. ( ) is treated with amoxycillin/clavulanic acid.
e. ( ) modified radical mastoidectomy is indicated in the
presence of complications.
7. Relative contraindications for cochlear implant include
a. ( ) cochlear nerve hypoplasia
b. ( ) incomplete partition Type 2
c. ( ) labyrinthitis ossificans
d. ( ) Michel deformity
e. ( ) otitis media with effusion

8. Meniere’s disease presents with


a. ( ) abnormal video head impulse test.
b. ( ) caloric abnormalities on vestibular testing.
c. ( ) episodic vertigo lasting for days.
d. ( ) oscillopsia.
e. ( ) persistent vertical nystagmus.

9. Peripheral nystagmus elicited during Dix Hallpike test


a. ( ) appears immediately as soon as the head is in a critical
position.
b. ( ) indicates pathology in the posterior semicircular canal.
c. ( ) is distinct from central nystagmus.
d. ( ) is in a fixed direction to the undermost ear.
e. ( ) last as long as the head position is maintained.

10. Regarding translabyrinthine approach for resection of vestibular


schwannoma
a. ( ) It allows facial nerve identification before tumor removal.
b. ( ) It is indicated in patient with non-serviceable hearing.
c. ( ) It is performed in tumor more than 5 cm.
d. ( ) It requires temporal lobe retraction.
e. ( ) There is less risk of cerebrospinal fluid leak compared to
rectosigmoid approach.

11. Malignant tumors of the temporal bone


a. ( ) has histopathology of adenocarcinoma in 60% of cases.
b. ( ) includes primary tumor arising from pinna.
c. ( ) is contraindicated to be treated surgically when the
infratemporal fossa is invaded.
d. ( ) is removed en bloc by a lateral temporal bone resection in
T3.
e. ( ) is staged as T4 according to Pittsburgh classification when
the tegmen is involved.

12. Regarding sudden onset sensorineural hearing loss


a. ( ) Computed tomography (CT) scan is indicated.
b. ( ) Intratympanic steroid is the first line of management.
c. ( ) It is defined as an average hearing loss of at least 40 dB.
d. ( ) Serological test for syphilis is required.
e. ( ) Test for diabetes is important in determining its choice of
treatment.

13. Regarding acoustic neuroma


a. ( ) 20% arises from the inferior vestibular nerve.
b. ( ) Excision via middle cranial fossa approach is indicated in
elderly patients.
c. ( ) Histologically shows both densely and loosely arranged
cells.
d. ( ) Is isointense on T2 weighted MRI.
14. In a patient who presents with sudden vertiginous attack,
a. ( ) a diagnosis of viral labyrinthitis is considered if hearing is
normal.
b. ( ) an MRI to rule out acoustic neuroma is indicated in majority
of cases.
c. ( ) perilymph leak is considered when associated with
fluctuating hearing loss.
d. ( ) presence of hearing loss is suggestive of vestibular neuritis.
e. ( ) responds to propanolol in vertiginous migraine.

15. Noise induced hearing loss,


a. ( ) exposure to noise more than 75 dB at least 6 hours a day is
a risk factor.
b. ( ) involves lower frequencies as exposure continues for longer
time.
c. ( ) PTA shows a dip at 4kHz.
d. ( ) transient threshold shift does not become permanent
threshold shift.
e. ( ) vitamin C is protective.

16. Hyperacusis
a. ( ) is a collapsed tolerance to usual environmental sound.
b. ( ) is due to paralysis of tensor tympani muscle.
c. ( ) is managed with conventional hearing aids.
d. ( ) is similar to recruitment.
e. ( ) the Loudness Discomfort Level (LDL) is below 85dB.

17. Tuning fork tests


a. ( ) A 128 Hz tuning fork is more commonly used for Rinne test.
b. ( ) A positive Rinne means air conduction is perceived louder
than bone conduction.
c. ( ) Stenger tuning fork test assesses which frequencies are
most affected by hearing loss.
d. ( ) Weber lateralises to the better cochlear.
e. ( ) Weber test is only effective if there is asymmetry of hearing
between the ears.

18. Bone anchored hearing aid (BAHA):


a. ( ) It is placed on the mastoid bone.
b. ( ) It is used for bilateral canal atresia.
c. ( ) It is used for single sided deafness.
d. ( ) It is used if the bone conduction hearing is better than 45 dB
e. ( ) The screw is made of platinum.

19. Audiological finding in Meniere disease include


a. ( ) directional preponderance on caloric test in the affected
side.
b. ( ) pure tone audiometry showing recruitment.
c. ( ) speech audiometry score of 100%.
d. ( ) the presence of tone decay.
e. ( ) type C tympanometry.

MAY 2019
1. Caloric test:
a. ( ) Irrigate ear at 30degrees for 40 sec T
b. ( ) Timed from onset of nystagmus F (from start of irrigation to
the end point of nystagmus is charted on calorigram)
c. ( ) Unilateral canal paresis means ipsilateral vestibular lesion T
(Less or no response of one canal indicates depressed function
of ipsilateral labyrinth, vestibular nerve, or vestibular cochlea)
NOV 2019
1. Carotid body tumour
a. ( ) Causes hypoglossal nerve palsy
b. ( ) Also known as chromaffin paraganglioma

2. Vagal paraganglioma
a. ( ) Neuroectodermal origin
b. ( ) Vagal causes horner’s syndrome
c. ( ) Involved nodus ganglion

3. Glomus tympanicum
a. ( ) Middle ear tumour can do LTBR

4. Tuning fork
a. ( ) Negative Rinnes need ABG of at least 20dB
b. ( ) Weber lateralise to better cochlea
c. ( ) Lower frequency tuning fork has shorter decay time
d. ( ) Stenger test
e. ( ) Rinnes negative in severe hearing loss

5. Middle ear implant


a. ( ) Attached to round window
b. ( ) Stimulate unilateral ear
c. ( ) Reduces occlusion effect
d. ( ) Can be done in children
e. ( ) Can use in CROS

6. Tinnitus
a. ( ) Measured by pitch matching
b. ( ) Tinnitus handicap index
c. ( ) Sound is masked with narrow band noise
d. ( ) Active distraction

7. Stapedial reflex
a. ( ) Hearing level 90dB can do
b. ( ) Tenses tensor tympani
c. ( ) Injury distal to stapedius will have absent reflex
d. ( ) Tympanometry A associated with absent reflex

8. Temporal bone trauma


a. ( ) Facial nerve in mastoid segment most common
b. ( ) CSF otorrhoea self limiting
c. ( ) Trautmann triangle posterior fossa
d. ( ) Otis capsule in longitudinal fracture
e. ( ) Presence of granulation tissue indicates of site of injury

9. Facial nerve palsy


a. ( ) Tympanic segment, dehiscence of canal is pathological
b. ( ) If there’s residual hearing, middle cranial fossa approach
c. ( ) Topodiagnostic
10. Benign bony lesion of ear
a. ( ) Exostosis is a true neoplasm
b. ( ) Osteoma is devoid of blood supply
c. ( ) Exostosis can be curetted

11. Nystagmus
a. ( ) Gaze evoked is peripheral
b. ( ) Spontaneous is abnormal
c. ( ) Congenital nystagmus is peripheral
d. ( ) Sedative will enhance Alexander’s law

12. CSF otorrhoea


a. ( ) Beta trace is diagnostic
b. ( ) Longitudinal fracture is more common
c. ( ) Most is self limiting

13. Caloric test


a. ( ) Cold water 30deg for 40 seconds
b. ( ) Position patient 30deg forward in order to put PSCC in
horizontal plane
c. ( ) Unilateral canal paresis is peripheral vertigo

14. Cholesteatoma
a. ( ) Canal cholesteatoma causes localised erosion
b. ( ) Kerastosis obturans is bilateral
c. ( ) Oral cavity Ca
d. ( ) HPV renders better survival

15. Masking
a. ( ) Bone conduction done using narrow band stimuli
b. ( ) Masking dilemma in bilateral symmetrical hearing loss

16. PTA
a. ( ) ABG less than 15dB normal
b. ( ) Air conduction tested up to 8kHx
c. ( ) Masking is done when air conduction between 2 ears is
40dB and above
d. ( ) SRT predicts PTA

17. Ear trauma


a. ( ) cauliflower ear
b. ( ) Commonly involves round window
c. ( ) Barotrauma in descent rather than ascent
d. ( ) TM perforation most heal spontaneously
e. ( ) Fluctuant hearing loss With vertigo is perilymph leak

18. Acoustic neuroma


a. ( ) reticular bleeds T
b. ( ) Sudden SNHL is a late presentation

19. Speech audiometry


a. ( ) Uses Pure tone of 40dB
b. ( ) In secretory otitis media get 100%
c. ( ) CSOM get 100%
d. ( ) Approaching 25dB in normal hearing
e. ( ) Reverse curve in vestibular schwannoma

20. Labyrinthine fistula


a. ( ) Exposed periosteum
b. ( ) Warrants exploration surgery
c. ( ) Tragal pressure positive nystagmus
d. ( ) Horizontal nystagmus stimulates P SCC
21. VEMP
a. ( ) Potential is high if S SCC dehiscence
b. ( ) Potential is low in BPPV
c. ( ) Potential absent in Ménière’s disease
d. ( ) Acoustic neuroma
e. ( ) Multiple sclerosis

22. Otosclerosis
a. ( ) Otic capsule
b. ( ) Autosomal recessive
c. ( ) Surgery is not justifiable in only hearing ear
d. ( ) Difficult to hear in loud noise
e. ( ) Commonly in posterior footplate

23. Cholesterol granuloma


a. ( ) There’s a cyst wall
b. ( ) Commonly at petrous apex
c. ( ) Histological diagnosis
d. ( ) Complete excision is required
e. ( ) HRCT temporal can differentiate between cholesteatoma
NOV 2020

1. Regarding coalescent mastoditis


a. Loss of septa separating mastoid air cells
b. Presents as acute mastoiditis
c. Treatment is mastoidectomy
d. Caused by prolonged course of antibiotic
e. Complicated with subperiosteal abscess

2. Bony mass in EAC


a. Osteoma is compact lamella bone
b. Exostosis can be easily curretted
c. Exostosis consist of cancellous bone histologically

3. Masking
a. Warble sound is used
b. Narrow band white noise used in air masking
c. Masking threshold is achieved when plateau is reached
d. Central masking is when the masked bone threshold shifts not more than
20dB threshold
e. Masking dilemma happens when sound to non test ear cross over to test ear

4. Meniere disease
a. Glycerol dehydration test improves hearing threshold
b. Intratympanic gentamicin a treatment
c. Vestibular neurectomy is destructive to hearing
d. Bilateral end stage disease is treated with cochlear implant
e. Turmarkin drop crisis is pathognomonic of Meniere's disease

5. Presbycusis
a. Metabolic is due to atrophy of stria vascularis
b. Cochlear type down slopping high frequency hearing loss
c. Cochlear have good speech descrimination score
d. Neural (cnt remember)
e. Treat with cochler implant

6. Vestibular schwanomma
a. Verocay body in Antoni A
b. Present with progressive unilateral hearing loss
c. Cochlear nerve invasion happens early
d. Treatment is by enucleation
e. Reticular type more difficult to treat

7. Cholesterol granuloma
a. Endothelial-lined cyst
b. Requires complete excision
c. Most common petrous apex lesion?
d. Able to differentiate with cholesteatoma with ct scan
8. Noise induced hearing loss
a. 80% of working adult exposed to NIHL
b. Characteristically has 4kHz notch in audiogram
c. Earmuff is adequate protection
d. Any exposure above 80dBHA requires ear protection
e. Noise at moderate frequency will cause reduction in blood flow

9. Temporal bone fracture


a. 80% vertigo
b. Sensorineural hearing loss in otic capsule sparing
c. Trapped epithelium at fracture line will cause cholesteatoma
d. Injury distal to chorda tympani absent stapedial reflex
e. ENOG within 3 days

10. Middle ear implant


a. Ideal for mixed hearing loss
b. Indicated for mild to severe snhl
c. Absent of Retrocochler hearing loss
d. Any Skin condition does not preclude

11. Otosclerosis
a. Notch at 4khz
b. Is treated with bone conduction hearing aid
c. Schwartze sign due to hyperaemia of tympanic membrane

12. Speech audiometry


a. Reach 100% at 25dB in normal hearing
b. SRT tested with spondee words
c. Reverse curve is seen in Retrocochler hearing loss
d. Tested at 40db?
e. Normal hearing is 100% at 25db

13. Auditory neuropathy


a. ABR is abnormal
b. OAE is normal
c. Brainstem implant is the treatment
d. Auditory dyssychrony and inner hair cells damage is included
e. Outer hair cells are affected

14. Acoustic reflex


a. Middle Brainstem lesion enables ipsilateral reflex
b. Coclear nucleus to both stapedius via contralateral superior olivary complex
c. Absent in Retrocochlear lesion
d. Present at 70 to 100 dBHL

15. Temporal lobe abscess


a. Auditory hallucination
b. Homonymous hemianopia
c. Intention tremor
d. Nystagmus
16. Facial nerve closely adjacent to these structures
a. Round window
b. Footplate of stapes
c. Processus cochleariformis
d. LSCC
e. ? Short process of incus

17. Cholesteatoma
a. Automastoidectomy results in stable epithelial lining
b. Preserve hearing with bridging of ossicular erosion
c. Bone erosion with osteoclastic enzyme
d. Matrix consist of keratinized squamous epithelium

18. Facial nerve palsy


a. Melkersson rosenthal syndrome = alternating facial nerve palsy
b. Bell's palsy is the commonest cause for recurrent facial nerve palsy
c. Herpes zoster oticus is variant of Ramsay hunt syndrome
d. Recovery in ramsay hunt syndrome 90%
ADDITIONAL SET
OTOLOGY AND NEURO-OTOLOGY
1. The following are true of conductive hearing loss
a. Lateralization in Weber’s test is present only when the
difference in hearing loss is more than 15 dB
b. In otosclerosis, Carhart’s notch is a depression of the air
conduction threshold around 2000 Hz
c. Ossicular chain damage in chronic otitis media with perforation
causes loss of 30 to 40 dB
d. The otoacoustic emission response can be used to estimate
conductive hearing loss
e. Ossicular discontinuity in an intact ear drum causes less of 50 –
55 dB

2. The following are true of ototoxic drugs


a. Salicylates cause reversible hearing loss
b. Streptomycin is used in the treatment of intractable Meniere’s
disease
c. Neomycin ototoxicity primarily affects the hair cells of the basal
turn
d. Gentamycin preferentially affects the vestibular system
e. Ethacrynic acid causes destruction of the stria vascularis

3. Cholesterol granuloma
a. Can be sequalae of temporal bone fracture
b. Presents with hyperintense signals in both T1 and T2 weighted
images on MRI
c. Commonly causes osteitis and resorption of bone
d. Results in cholesteatoma formation in chronic cases
e. Requires complete exenteration of mastoid cells during
mastoidectomy to prevent recurrence

4. The following are true of masking:


a. Masking is a procedure to isolate the left and right ear during
audiometry
b. Masking is presented by air conduction whether testing is of air
or bone conduction
c. Air conduction signals cross over at 50dB
d. Insert receivers increase interaural attenuation to 70 dB to 80dB
e. Bilateral 50dB air-bone gaps create a masking dilemma

5. The following are true of the cochlea:


a. The helicotrema is apical communication of scala tympani and
scala media
b. The cochlear duct is connected to the saccule via the ductus
reunions
c. The limbus is a thickened periosteum of the osseous spiral
lamina
d. The Organ of Corti in man, consist of about 12, 000 inner hair
cells and 3,500 outer hair cells
e. The tectorial membrane is a gelatinous leaf which stretches from
the limbus to the stria vascularis
6. The following are true of the blood supply of to the middle ear
a. The lenticular process commonly undergoes necrosis when
there is a fracture of the long process of the incus
b. The subarcuate artery is usually a branch of the labyrinthine
artery
c. The inferior tympanic artery plays a major role in providing blood
supply to glomus tympanicum
d. The persistent stapedial artery represents the residua of the
embryonic hyoid artery
e. The stylomastoid artery supplies the facial nerve fibers in
fallopian canal

7. The following are true of the management of traumatic facial nerve palsy
a. Lesion of the facial nerve proximal of the geniculate ganglion
causes ipsilateral loss of stapedial reflex
b. Topodiagnostic test give a precise indication of a site when the
lesion is partial
c. Electroneurography demonstrating greater than 90% neural
degeneration within the first 6 days of temporal bone fracture
suggests the need for surgical exploration
d. When the hearing is normal, the appropriate approach for
decompression at the labyrinthine segment is via a transmastoid
approach
e. The middle cranial fossa approach is indicated when there is
absent of lacrimation with complete sensorineural hearing loss

8. The following are true of caloric test


a. The head is raised 45 degree above horizontal
b. Cold stimulation of the labyrinth induces a nystagmus with the
slow component to the left
c. Directional preponderance of nystagmus only indicates a
peripheral nystagmus
d. Only the lateral semi-circular canals are stimulated
e. Frenzel glasses is applied at the cessation of nystagmus

9. The following are true of subjective tinnitus


a. Is present in a functionally dead ear
b. It is masked by wide band noise
c. Hearing aid is useful for patient suffering from tinnitus and
hearing loss
d. In patients with cochlear disorder, section of the eight nerve
does not necessarily affect the quality of tinnitus
e. Aspirin can temporarily make the tinnitus worse

10. The following are true of facial nerve


a. The motor fibers arise from the precentral gyrus
b. Its bony canal dehiscent at the horizontal portion in 50% of the
cases
c. The chorda tympani branch off more proximal then the nerve to
the stapedius
d. The greater superficial petrosal nerve joins at the facial hiatus
e. The peak to peak amplitude in electroneuronography is
indirectly proportional to the number of intact motor axons
11. The following are true of Meniere’s Disease
a. Vestibular neurectomy a more effective treatment than saccule
decompression
b. It is characterized by an abnormally small negative summating
potential in electrocochleography
c. Betahistine chloride is a labyrinthine sedative
d. Medical treatment usually provides adequate control of
symptoms in 80% of patients
e. Labyrinthectomy is an option for patients with intractable
disease with profound hearing loss

12. The following are true of Brainstem Evoked Response Audiometry


a. Delayed wave I latency can be due to conductive hearing loss
b. Interaural wave V latency delay of more then 0.4 msec is
suggestive of acoustic neuroma
c. Delayed wave I to V latency can occur in infants below 18 month
of age
d. At high click stimulus intensity of 85 dB , wave V latency is
normally about 34.5 msec
e. Wave V is due to response at the superior colliculus

13. The following are true of Eustachian tube


a. It is about 25mm in length in adults
b. It is shorter and wider in infants
c. Has a pharyngeal cartilainou part which froms two-thirds of its
length
d. The internal carotid lies lateral to the bony portion of the tube
e. It closes with contrction of the tensor palati muscle

14. The following are true in the management of cholesteatoma


a. Cholesteatoma causing labyrinthine fistula has a positive fistula
test in only of about 30% of cases
b. Cholesteatoma limited to the attic or antrum is approached via
atticotomy
c. Exteriorization of cholesteatoma can halt bone resorption even
without complete removal
d. Canal wall up procedure is recommended in poorly pneumatized
mastoids
e. Cholesteatoma in the sinus tympani is adequately removed by
performing a posterior tympanotomy

15. The following are true of lower motor neuron facial nerve injury
a. Decompression is indicated when there is more than 90%
degeneration on electroneuronography
b. Injury at the labyrinthine segment will have normal gusto
manometry results
c. Polyphasic potentials in electromyography indicates
reinnervation
d. Loud tone stimulation in the contralateral ear does not produce
a change in impedance if the injury is proximal to the nerve to
the stapedius
e. If associated with profound to severe hearing loss , the best
approach to the labyrinthine segment is via the middle cranial
fossa approach
16. The following criteria suggest vestibular neuronitis
a. Sudden onset of vertigo
b. Mild to moderate hearing loss
c. Vertigo lasting 4-5 days
d. Occurs predominantly in the elderly
e. Decreased ENG caloric response in one ear

17. Grommets
a. Are inserted mainly to drain fluid from the middle ear
b. Allow middle ear to be ventilated normally
c. Are very useful in Meniere’s disease
d. With recurrent infections can cause changes in the tympanic
membrane
e. Must be removed surgically after 6b months

18. The following are true of perilymph fistula


a. Explosive mechanism involves a sudden increase in middle ear
pressure causing a fistula
b. A positive Hennebert’s sign confirms the diagnosis
c. Recurrent meningitis is a presenting symptom in children
d. The round window is recognized as a route of perilymph leakage
e. Surgical exploration is indicated after failure of conservative
management for 36-48 hours

19. The following are true of stapes surgery


a. The fat and wire prosthesis is preferred in small fenestra surgery
b. The facial nerve is found inferior to the footplate
c. Fracture of the long process of the incus is a delayed
complication
d. Reparative granuloma occurs between 10th to 15th
post-operative days
e. Presence of round window otosclerosis requires drilling of the
obliterative bone

OTOLOGY
1 F F T F T
2 TTTT T
3 T T T F F
4 T T F T T
5 FTTFF
6 TTTT F
7 TFTF F
8 FTFT T
9 TTTT T
10 TTFT F
11 TFFT T
12 TTTF F
13 FTTF F
14 TTTF F
15 TFTT F
16 TFTF T
17 FTFT F
18 FFTT F
19 FFFTF
USM COLLECTION
AUDIOLOGY AND OTOLOGY

1. Which of the following statement are true in vertigo


a. In Meniere’s disease the duration of vertigo lasted for few
seconds
b. One of the central characteristic of spontaneous nystagmus is
vertical upward nystagmus
c. Spontaneous nystagmus is always pathological
d. A fixation nystagmus always indicate central lesions
e. Romberg positive indicate recent development of a unilateral
vestibular disorder
FTFTT

2. Betaserc ( betahistine hydrochloride)


a. Is a histamine analogue that acts as H3- antagonist
b. Is used for fast relief of vertiginous symptoms
c. Should be used in precautions in patient with bronchial asthma
d. Available in 8mg, 16mg, and 48mg preparation
e. It accelerates the blood circulation in the inner ear at
microvascular level
TFTFT

3. Regarding fistula test


a. It is used to diagnose the presence of fistula in the perilymphatic
duct
b. Hennebert’s sign indicate a false positive fistula test
c. False negative results can indicate a dead labyrinth
d. In the presence of fistula at the lateral semi-circular canal, the
deviation of eyes is towards the normal ear
e. The application of air pressure changes into the middle ear can
be done by means of Siegelization using a pneumatic otoscope
FTTTT

4. Acoustic neuroma
a. Is a benign tumour involving cells of the myelin sheath of he
vestibulocochlear nerve
b. Unilateral acoustic neuroma is associated with
Neurofibromatosis Type 1
c. Bilateral acoustic neuroma is associated with Neurofibromatosis
Type 2
d. Can occur anywhere along the vestibular nerve pathway
e. Facial pain in acoustic neuroma indicates compression of the
facial nerve
TFTTF

5. Hearing assessment in acoustic neuroma


a. PTA may show an asymmetrical SNHL
b. There is good speech discrimination
c. The hearing loss is classically a neural lesion with recruitment
d. BSER showed an increased latency between N1 and N5 waves
e. BSER has 3 % of false negative
TFFTT
6. In tuning fork test
a. The inner ear are normally twice as sensitive to sound conducted
by bone as to that conducted by air
b. The lower frequency tuning fork has longer decay time and
higher frequency tuning fork
c. Rinne positive is obtained from normal ears and unilateral
sensorineural loss
d. Rinne negative is obtained from a conductive hearing loss in
excess of 15-20dB HL
e. Rinne negative is obtained in cases of severe or total
sensorineural hearing loss in the test ear
FTTTT

7. In impedance audiometry
a. The dynamic changes measure the stapedius muscle contraction
b. Tympanometry measures the compliance of the tympanic
membrane
c. In acoustic reflex measurements the minimum sound intensity
required to produce response is 60dB above the normal
threshold
d. The afferent limb of the reflex arc consists of the CN VII,
stapedius muscle and tendons and stapes
e. A normal contralateral stapedius reflex implies an intact afferent
limb of the reflex arc on the side of acoustic stimulus
TTFFT

8. The following variable s may affect the acoustic reflex measurement


a. Presence of ipsilateral minimal conductive hearing loss of 5-
10dB
b. Presence of mild cochlear impairment of 20-40dB
c. Presence of CN VII dysfunction
d. Presence of mild CN VIII impairment of 0-40 dB
e. Presence of tympanic membrane perforation
TFTT?

9. Regarding facial nerve


a. The nerve enters the temporal bone through the porus acoustic
and internal auditory canal
b. 50% of the dehiscent facial canal occurs in vertical part
c. Facial nerve injury is more likely to occur in transverse type of
temporal bone fracture compare to longitudinal fracture
d. In longitudinal temporal bone fracture, injury to the facial nerve
is usually due to compression rather than severance of the nerve
e. The chorda tympani branched off from the main trunk distal to
stylomastoid foramen
TFTTF
10. The following are true
a. High facial ridge is the most common cause of persistent
otorrhea in post mastoidectomy patient
b. The advantage of using diamond-paste burr in mastoidectomy
include stopping haemorrhage from bone
c. In modified radical mastoidectomy the incus, malleus and stapes
are removed
d. Radical mastoidectomy includes removal of the handle of
malleus and TM remnant
e. The long process of incus is a good landmark for the lateral semi-
circular canal
TTFTF

11. The following CT scan findings should increase suspicion of temporal


bone fracture
a. Air fluid level in the middle ear of the fracture side
b. Air fluid level in the mastoid air cells of the fracture side
c. Opacification of the mastoid air cells of the fracture side
d. Intracranial air within the subarachnoid space of the
cerebellopontine angle
e. Air fluid level in frontal sinus
TFTTF

12. Regarding recruitment


a. Occurs in the presence of cochlear lesion in particular the
damage in the inner hair cells
b. Occurs in the presence of retrocochlear lesion
c. A patient with SNHL with recruitment in one ear has a reduced
dynamic range in the affected ear
d. In the presence of recruitment , loudness discomfort levels is
reduced as compared to the normal population
e. Loudness recruitment is absent in disorder affecting the auditory
division of the CN VIII
FFTFT

13. The following are true


a. The principal blood supply of the facial nerve is from the
stylomastoid artery
b. The meatal segment within the internal auditory canal has the
poorest blood supply
c. Most of the injury to the facial nerve occur at the horizontal
segment in longitudinal fracture of the temporal bone
d. Both conductive hearing loss and high frequency sensorineural
hearing loss can occur in longitudinal fracture of temporal bone
e. The onset of facial nerve palsy is often delayed in transverse
fracture of the temporal bone fracture
TFFTF

14. The following are true


a. Patients with Cogan syndrome experience episodic vertigo
b. In Wallenberg syndrome, patient has episodic vertigo
c. In lateral medullary syndrome patient develop dysphagia
secondary to ipsilateral palatal paresis
d. The underlying pathophysiology in Wallenberg syndrome is
embolic event of ipsilateral vertebral artery
e. Cogan syndrome is characterised by interstitial keratitis
TFTTT
15. The following autoimmune disease(s) is/ are associated with hearing loss
a. Behcet’s disease
b. Polyarteritis nodosa
c. Cogan syndrome
d. Temporal arteritis
e. Sarcoidosis
TTTTF

16. The following are topodiagnostic test of facial nerve injury secondary to
temporal bone fracture
a. Pure tone audiometry
b. Stapedial reflex
c. Salivary flow test
d. Schimmer’s test
e. Electroneuronography
FTTTF

17. The following is are true of otosclerosis


a. Males are more commonly affected
b. Pregnancy worsened the symptoms
c. Associated with alpha-1-antitripsin deficiency
d. A dip at 2KHz in air conduction is known as Carhart’s Notch
e. Can causes SNHL
FTTFT

18. Which of the following is/ are true


a. The tympanic ring continues to expand and thicken into adult life
b. The primary auditory cortex corresponds to Broddmann’s area
21
c. Cerumen is bacteriostatic
d. The narrowest portion of the membranous labyrinth is the
ductus reunions between the cochlea and utricle
e. Eustachian tube is opened at rest
TFTFF

19. The following are hearing aids components


a. Earmold
b. Power supply
c. Amplifier
d. Receiver
e. FM transmitter
TTTTF

20. Regarding CT scan in temporal bone fracture


a. Axial CT scan are most useful in delineating transverse fracture
b. Coronal CT scan are most useful in delineating longitudinal
fracture
c. In high resolution CT scan , 1-2mm thickness cut is required for
better facial nerve injury assessment
d. Details of the temporal bone fracture are best visualized when
the plane of tomographic sectioning is at 90 degree to the plane
of the fracture
e. Presence of air fluid level in the middle ear in CT scan raise the
suspicion of temporal bone fracture
FFTTT
21. The following is /are true
a. Tonsil of Gerlach is referred to lymphoid tissue within the torus
tubarious
b. Korner’s septum separates the squamous from the petrous air
cells
c. Tympanic plexus is partly formed by CNVIII
d. The lateral process of the malleus forms the inferior boundary of
Prussak’s space
e. The tympanic portion of CNVII is dehiscent in 5% of population
FTFTF

22. The impedance- matching system of the middle ear


a. Only the lower two-thirds of the tympanic membrane is
effectively vibrating
b. Movements of tympanic membrane are more at the periphery
than at the centre
c. The handle of malleus is 1.3 times longer than the short process
of the incus
d. Converts sound of greater amplitude to lesser amplitude
e. The area effects of the tympanic membrane is in the ratio of 17:1
to the stapes footplate
TTFTT

23. With regards to facial nerve injury


a. Electromyography will be benefits in early onset of facial nerve
paralysis
b. Electromyography is good in assessing the prognosis in long
standing facial paralysis
c. Electromyography is particularly helpful early in the course of
traumatic facial paralysis
d. In infant the facial nerve is well protected by the development
of mastoid process
e. Neuropraxia can be distinguished from complete avulsion by
electrophysiologic studies
FTTFT
HEAD & NECK
NOV 2007

1. Investigation to diagnose Sjogren syndrome


a. FBC
b. Rheumatoid factor
c. Sublabial biopsy
d. Liver function test
e. ANA
FTTFT
2. Thyroid gland
a. Joll’s triangle is for recurrent laryngeal nerve identified
b. Non recurrent laryngeal nerve is at left side
c. Recurrent laryngeal in danger when cutting in membrane of the
thyroid
d. Berry’s ligament is attachment of the thyroid to trachea
e. Superior thyroid artery gives supply to lateral wall of the thyroid
gland
FF FTT

3. In head and neck


a. NG tube needed for fedding 7-10 days with laryngectomy
operation
b. Drains are removed in the 3rd day post op
TF

4. Medullary ca
a. We excised the parathyroid gland together with it
b. Hypercalcaemia is a feature
c. Common in female
TFT

5. Salivary gland
a. Minor gland more prone for malignancy
b. Rhabdomyosarcoma is aggressive tumour in this area
c. Get innervations from otic ganglion
TTF

6. At the mandible
a. Ameloblastoma is a fast-growing tumour
b. Pre-canine is the commonest type of odontogenic cyst
c. At the mandible odontogenic cyst are radiolucent
d. Recurrences are high with the odontogenic cyst
e. In the odontogenic cyst marsupialization is one of the
treatments
FFTFT

24. Infectious mononucleosis


a. Is cause by EBV
b. Associate with hearing loss

TT
MAY 2008
1. Post op mx of H&N surgery
a. Oral feeding is started in 7-10 days in laryngectomy patient
b. Drain should be removed 3 rdday op
c. Serum calcium is done next day
d. Se thyroid is done the next day
e. In gastric pull up patient calcium supplement is needed
TFTFF

2. In DIfferentiated thyroid cancer


a. A papillary is common in female
b. Follicular is easily diagnosed in FNAC
c. Anaplastic histology is similar to lymphoma
d. Lymphoma Is part of DTC
e. Medullary is familial
TFTFT

3. Branchial cyst
a. Is same as branchial sinus
b. Is found in parotid
c. Inclusion theory explained the internal opening
d. The swelling is intermittent
e. It present since birth
TTFTF

4. Parotid tumour
a. Pleomorphic adenoma is multicentric
b. Whartins is bilateral
c. Open biopsy is absolute contraindication
d. In retrograde to search facial nerve use cervical branch
TTFF

5. In neck dissection
a. In supraomohyoid level I & II are removed
b. Lateral neck dissection level I is not removed
c. SCM is preserved in modified type III
FTT

6. Contraindication of RND
a. Supraclavicular LN is contraindication
b. Untreatable primary disease
c. Liver metastasis
d. Carotid sheath involvement
e. Recent MI
FTTTT

8. Corners of consternation
a. Upper end of IJV
b. Submandibular triangle
c. Lower end of omohyoid
d. Lower end IJV
e. Supraclavicular triangle
TTFTF
9. Fracture of mandible
a. The weakest is the subcondylar region
b. Direction of the fracture is determine by musculature
c. If there is bone loss, we use open pin method
d. Commonest side of mandibular fracture is condylar
e. Teeth at the line of fracture must be removed
TTTTF

10. Diagnosis of Sjogren’s syndrome


a. FBC
b. LFT
c. Rheumatoid factor
d. Serum folate
e. Sublabial biopsy
FFTFT

11. Component of polysomnography


a. Electroencephalogram
b. ECG
c. ENG
d. Pulse oximetry
e. EOG
TTFTT

12. Pedicled myocutaneous flap


a. Trapezius
b. Rectus abdominis
c. Deltopectoral
d. Forehead
e. Latissimus dorsi
TFTFT

13. Skin graft


a. Is used to cover mandible
b. Full thickness included the entire dermis
c. Split thickness ……
d. In full thickness, no need immobilization

FT?F
NOV 2008

1. Which of the following statement are true?


a. Marrow invasion of the mandible is better identified by CT than
MRI
b. MRI detects perineural tumour extension better than CT
c. Ultrasound is the most reliable morphology imaging technique
for the thyroid gland in the neck
d. Ultrasound more reliably detects parathyroid adenoma in the
neck than radionuclide scanning
e. Ultrasound guided fine needle aspiration allows simple
diagnostic intervention for evaluation of neck lesions smaller
than 1 cm
FTTTF

2. A patient is noted to have ptosis of the ipsilateral upper eyelid following


a radical neck dissection. This finding is due to inadvertent dissection
deep to
a. Internal jugular vein
b. Common carotid artery
c. Brachial plexus
d. Omohyoid muscle
e. Phrenic nerve
?TFFF
3. The following are contraindications for free tissue transfer in head and
neck reconstruction
a. Age greater than 80 years
b. Age less than 10 years
c. Raynaud’s phenomenon
d. Obesity
e. Donor site irradiation
FFT?T
4. With regards to parapharyngeal tumours
a. Most are neurogenic in origin
b. Diagnosis is confirmed by FNAC or open biopsy
c. Most common salivary gland tumour is mucoepidermoid CA
d. Most parapharyngeal neurogenic tumours are vagal
schwannomas
e. Vagal schwannomas produces vocal cord palsy
TFFTT

5. With regards to acute tonsillitis


a. A preceding viral infection of the upper respiratory infection is a
predisposing factor
b. In infectious mononucleosis, the absolute lymphocyte count is
reduced
c. It may be associated with carbimazole
d. Streptococcus pyogene is the commonest bacterial cause
e. Mitral valve stenosis is one of late complication

TFTTT
6. The following statement regarding obstructive sleep apnoea syndrome
are true?
a. The lowest oxygen saturation is an indication of its severity
b. Uvulopalatopharyngoplasty is commonly complicated by
changes in voice
c. An apnoea-hypopnoea index of 20 indicates severe sleep
apnoea
d. Retrognathia is a predisposing cause
e. Hypertension is a known association
TFFTT

7. Regarding mandibular cyst


a. The follicular cyst is radiolucent
b. The periapical cyst is the most common type
c. Calcifying odontogenc cyst is also known as Gorlin cyst
d. Odontogenic keratocyst has an aggressive clinical behaviour
with high recurrence rate
e. Enucleation is the treatment of choice in odontogenic keratocyst
TTTTT
MAY 2013
1. Thyroid ca
a. ( ) Monoclonal origin
b. ( ) RET proto-oncogene is anaplastic
c. ( ) Transmembrane thyroxine kinase receptor reduced in
papillary carcinoma
d. ( ) P53 not rare
e. ( ) Medullary ca from parathyroid

2. Fibrous dysplasia of mandible


a. ( ) Mandibulectomy is the treatment of choice
b. ( ) Loss of distinction of inferior mandibular cortex
c. ( ) Growth spurt which subsides after puberty
d. ( ) Bony hard swelling
e. ( ) Radiographic appearance of ground glass trabecular pattern
f. ( ) Radiographic appearance of sharply demarcated lesion

3. Chemotherapy
a. ( ) Carboplastin more nephrotoxicity than cisplastin
b. ( ) Cisplastin is associated with ototoxicity
c. ( ) Combination chemotherapy gives good result in recurrent
malignant tumours of H&N
d. ( ) Gene therapy uses viral vector
e. ( ) Can be used in treatment of metastatic lymphoepithelioma

4. Pyriform sinus ca
a. ( ) Spread to larynx via paraglottic space
b. ( ) Continuous with neck secondaries
c. ( ) Ear pain is early sign
d. ( ) Primary tumour can present in the neck through the
thyrohyoid membrane

5. True regarding hypopharyngeal ca


a. ( ) EGFR overexpression
b. ( ) Associated with Plummer Vinson in pyriform fossa ca
c. ( ) Submucosal spread is seen in superior hypopharyngeal
tumours
d. ( ) 50% involve RLN
e. ( ) 50% vocal cord palsy

6. True regarding odontogenic tumour


a. ( ) Gardner syndrome is associated with intestinal polyps
b. ( ) Dentigerous cyst wall contain ameloblastoma
c. ( ) Ameloblastoma multicystic
d. ( ) Ameloblastoma - tooth root resorption
MAY 2014
1. Echelon nodes
a. ( ) Subglottic – level III
b. ( ) Oropharynx – II, III, IV
c. ( ) Scalp behind the level of tragal – preaucricular, suboccipital,
post- auricular
d. ( ) Oral cavity – II, III, IV

2. Thyroidectomy
a. ( ) Parathyroid gland can be preserved in MNG
b. ( ) Joll’s triangle is associated with RLN
c. ( ) Inferior thyroid artery cross anterior to internal laryngeal
nerve
d. ( ) Superior thyroid artery is ligated near to the superior pole
of the gland

3. Basal cell carcinoma


a. ( ) Locally invasive
b. ( ) Not sensitive to radiotherapy
c. ( ) 5mm surgical margin is adequate
d. ( ) Rarely metastasize
e. ……………………………………….

4. Suppurative parotitis
a. ( ) Commonest pathogen is staph aureus
b. ( ) More common in children between 6-10 years old
c. ( ) If recurrence, need superficial parotidectomy
d. ( ) Associated with xerostomia
e. ( ) Can be complicated with Vincent angina

5. True regarding pleomorphic adenoma


a. ( ) It contained epithelium and mesodermal
b. ( ) The genetic causing it had been identified
c. ( ) Oncocytoma is encapsulated
d. ( ) Warthin’s is exclusive in submandibular
MAY 2015
1. Base of tongue carcinoma
a. ( ) Vallecula involvement is indicated for total glossectomy
with laryngectomy
b. ( ) Transoral approach surgery is indicated for mobile tongue
c. ( ) Total glossectomy is indicated when genioglossus is
involved
d. ( ) Lymphatic drainage to retropharyngeal nodes
e. ( ) Occult metastasis is rare

2. Thyroglossal cyst
a. ( ) Most common at level of thyroid
b. ( ) Risk of carcinoma is common
c. ( ) Removal include removal of hyoid bone
d. ( ) Differential is dermoid cyst

3. OSA
a. ( ) Cicatricial scarring is a complication of UPPP
b. ( ) Bana ? surgery is option for severe OSA
c. ( ) Mueller manoeuvre to grade severity of OSA
d. ( ) Base of tongue reduction of retropalatal collapse
e. ( ) Hyoid suspension for retrolingual collapse

4. Follicular thyroid ca
a. ( ) Form the majority of thyroid malignancies
b. ( ) Present as single solitary nodule
c. ( ) Metastasize to lung more likely compare to lymph node
d. ( ) Cannot be differentiated from adenine in FNAC
e. ( ) Is a malignant transformation from thyroglossal cyst

5. Lip carcinoma
a. ( ) Most common occur in the upper lip
b. ( ) Is staged according to oral cavity ca
c. ( ) Show basal cell carcinoma as commonest pathology
d. ( ) Show Clinical behaviour of skin carcinoma
e. ( ) Is directly related to amount of Sun exposure

6. Branchial cyst
a. ( ) Most a/w internal opening
b. ( ) Malignant transformation is common
c. ( ) Contain cholesteral in the cystic fluid
d. ( ) Peak incidence during childhood
e. ( ) Cystic degeneration of lymph nodes

7. Regarding grading of salivary gland


a. ( ) Acinic cell – low grade
b. ( ) Myoepithelial ca – high grade
c. ( ) Adenocystic ca – high grade
d. ( ) Papillary cystadenocarcinoma – low grade
e. ( ) Squamous cell ca – high grade
f. ( ) Undiferreniated ca – high grade

8. Angle of mandible mass different from parotid mass including


a. ( ) Papillary cystadenoma
b. ( ) HIV related cyst
c. ( ) Hydatid cyst
d. ( ) Sjogren
e. ( ) Sarcoidosis
8. Parapharyngeal mass
a. ( ) Biopsy is best taken intra orally
b. ( ) Mass will push lateral pharyngeal wall, cause medialization
c. ( ) Schwannoma cause splaying of the carotid
d. ( ) Mass at post styloid push carotid anterolateral
e. ( ) Vagal schwannoma splay the internal and external carotid
artery
f. Referred otalgia seen in glomus vagale is due to irritation of
pharyngeal plexus

9. Regarding hypopharyngeal tumour


a. ( ) Pooling of saliva in the pyriform fossa is found
b. ( ) Edema of the aryhteniod is suggestive feature
c. ( ) Potassium deficiency is a feature
d. ( ) Trans oral CO2 laser is indicated for adequate tumour
e. ( ) Involvement of prevertebral fascia indicated inoperability

10. Trauma OMFS


a. ( ) Tear drop sign in orbital floor fracture
NOV 2015
1. Dentigerous cyst
a. ( ) From infected tooth
b. ( ) Radicular cyst cause thinning floor of maxillary sinus
c. ( ) Common in maxilla
d. ( ) Odontogenic keratosis arise from unerupted tooth
e. ( ) Ameloblastoma treat with radiotherapy

2. OSA
a. ( ) High occurrence in stage 1 sleep
b. ( ) Outcome surgery depend on obesity (BMI)
c. ( ) Frequently in REM sleep

3. Pleomorphic adenoma
a. ( ) Most common in parotid
b. ( ) Treat with resection include normal tissue
c. ( ) Presence of true capsule
d. ( ) Risk of malignancy
e. ( ) If recurrence of surgery

4. Parotid ca
a. ( ) Adenoid cystic ca is one third of all major salivary gland
b. ( ) Perineural invasion
c. ( ) Lung metastases
d. ( ) Immunotherapy is not effective

5. Reconstruction of head and neck


a. ( ) Pectoralis major is a free flap
b. ( ) Bare cartilage cover with SSG
c. ( ) Deltopectorial flap blood supply is acromiothoracic artery
d. ( ) Allen test for radial forearm flap
e. ( ) Jejunal loop for post cricoid ca

6. Thyroglossal duct cyst


a. ( ) Invade thyroid cartilage
b. ( ) Between foramen cecum and thyroid gland
c. ( ) Treatment is cystectomy
d. ( ) Move with swallowing

7. Risk of non melanoma ca


a. ( ) Renal transplant
b. ( ) Young age
c. ( ) Fair skin
d. ( ) Xeroderma pigmentation
e. ( ) Isotretinoin

8. Tonsillar hypertrophy causes


a. ( ) LPR
b. ( ) Lymphoma
c. ( ) Quinsy
d. ( ) Infectious mononucleosis

9. Outcome of ca hypopharynx depends on


a. ( ) Lymph node metastases
b. ( ) Age
c. ( ) Larynx involvement
d. ( ) Sideroblastic anaemia
10. Regarding neck dissection
a. ( ) T1 N0 in oral ca require supraomohyoid
b. ( ) Lymph node in IIB is easily identified
c. ( ) Parotid ca requires anterolateral neck dissection
d. ( ) Phrenic nerve can be injured in MRND
e. ( ) Transverse process of atlas is surgical landmark

11. Thyroid neoplasm


a. ( ) Medullary type is autosomal dominant
b. ( ) Papillary most common
c. ( ) Nodal metastases is bad prognosis
d. ( ) Young age is a risk factor

12. Ca of oral cavity


a. ( ) Buccal ca most common
b. ( ) Betel nut chewing is a risk factor
c. ( ) Tooth decay is a risk
d. ( ) T3 tongue ca is transoral approach
e. ( ) Spindle cell treat with …
f. ( ) Early stage can treat with radiotherapy
g. ( ) Ca tongue in situ require neck dissection
MAY 2016
1. Chyle leak
a. ( ) Occurs in 30% of neck dissection on the left side
b. ( ) Indication for neck exploration if the output is more than
600ml/day
c. ( ) Recurrent aspiration is advocated
d. ( ) Vacuum dressin is part of the treatment
e. ( ) Likely to damage the phrenic nerve if left untreated

2. Regarding ameloblastoma
a. ( ) Associated with unerupted third molars
b. ( ) 80% occurs in maxilla
c. ( ) Radiographically is a multicystic lesion with tooth
resorption
d. ( ) Multicystic type require hemimandibulectomy and
reconstructive
e. ( ) Commonly metastasize to lung

3. Buccal ca
a. ( ) F>M , 2: 1
b. ( ) Orogenital contact is a risk
c. ( ) Secondary healing for T1 lesion
d. ( ) Photodynamic therapy for superficial lesion
e. ( ) HPV is associated with SCC

4. Complication of parotidectomy
a. ( ) Gustatory flushing
b. ( ) Negative starch iodine test
c. ( ) Numbness at inferior ½ of pinna

5. OSA
a. ( ) PSG somnoplasty done 3/12 after op
b. ( ) Radiofrequency tongue base volume reduction (RFTVR) ??
c. ( ) 50% recurrence of UPPP
d. ( ) Fujita type II indicate retrolingual collapse
e. ( ) Morbid obesity best treated with bariatric surgery

6. Thyroglossal duct cyst


a. ( ) Occur at birth
b. ( ) Diagnosis is confirmed by FNAC
c. ( ) Fistula is surgical complication
d. ( ) Remove body of hyoid

7. Fibrous dysplasia
a. ( ) ALP increase in polyostotic
b. ( ) More in maxilla
c. ( ) More involve adult
d. ( ) Fibroosseous tissue fill up cortex
e. ( ) Extracranial present more with pain

8. Hypopharyngeal carcinoma
a. ( ) T3 is hemilarynx fixation
b. ( ) Pyriform sinus present earlier than post cricoid carcinoma
c. ( ) Postcricoid ca female > male
d. ( ) Pyriform fossa ca in Plummer -Vinson syndrome
e. ( ) Laser can be used for early lesion
NOV 2016
1. Radio iodine therapy in differentiated thyroid carcinoma
a. ( ) Induced secondary malignancy in minority of patients
b. ( ) Increase risk of miscarriage
c. ( ) Not useful in medullary thyroid ca
d. ( ) Uses iodine-131
e. ( ) Commonly causes pulmonary fibrosis

2. Thyroglossal duct cyst


a. ( ) Commonest midline neck swelling
b. ( ) Lingual thyroid is a differential diagnosis
c. ( ) Radioiodine scan done in the absence of normal thyroid
gland
d. ( ) Excision associated with higher recurrence rate
e. ( ) Opens into foramen caecum

3. Approach to oropharyngeal mass/ oropharynx


a. ( ) Lateral mandibulotomy
b. ( ) Paramedian mandibulotomy
c. ( ) Lateral pharyngotomy
d. ( ) Lingual release
e. ( ) Transcervical transparotid

4. Hypopharyngeal ca
a. ( ) T2 best treated with radiotherapy
b. ( ) Pharyngolaryngectomy is performed in Post cricoid tumour
c. ( ) Jejunal flap can be used for reconstruction
d. ( ) Supraomohyoid neck dissection is performed in N0 neck
e. ( ) Loss of laryngeal crepitus indicates extension of tumour to
prevertebral muscles

5. Regarding thyroid and parathyroid surgical anatomy


a. ( ) Joll’s (sternothyrolaryngeal) triangle is associated with the
external branch of superior laryngeal nerve
b. ( ) Recurrent laryngeal nerve is the content of Beahrs triangle
c. ( ) Inferior thyroid artery supply the superior parathyroid
gland
d. ( ) Non recurrent laryngeal nerveis found on the left side
e. ( ) The isthmus is crossed over by the anterior jugular veins

6. Regarding radiotherapy
a. ( ) Cause single strand DNA break
b. ( ) Effect is maximum at mitotic phase
c. ( ) Causes minimal complication in rapidly dividing cells
d. ( ) Action is optimized in well oxygenated cell/ tissue
e. ( ) Fractionation causes less side effects

8. Zenkers diverticulum / pharyngeal pouch


a. ( ) Origins in between middle constrictor muscle and inferior
constrictor muscle
b. ( ) Barium swallow findings is diagnostic
c. ( ) The malignant transformation is rare
d. ( ) Dohlman procedure is an endoscopic excision approach of
the pouch
e. ( ) Pathophysiology of the lesion includes spasm of
cricopharyngeus muscle
9. Paranganglioma
a. ( ) Arise from chromaffin neural crest
b. ( ) In paraganglioma that confine to ME , lateral approach is
and option
c. ( ) Involvement of hypoglossal suggestive of glomus jugulare

10. Submandibulectomy
a. ( ) Dissection is Subcapsular
b. ( ) The marginal mandibular nerve encountered superficial to
platysma
c. ( ) Hypoglossal nerve is at risk
d. ( ) Facial artery is an important landmark
e. ( ) The duct ligated distally

11. Dentigerous cyst


a. ( ) Present as well defined Unilocular radiolucency in xray
b. ( ) Arise from dental follicle of unerupted tooth
c. ( ) Peak incidence during teenage years
d. ( ) Usually asymptomatic
e. ( ) It is a type of odontogenic cyst

12. Regarding cervical occult primary


a. ( ) Primary site is found in 2/3 of the case
b. ( ) Commonest primary is base of tongue
c. ( ) Selective neck dissection is the best surgical option
d. ( ) Excision is first step in management
e. ( ) SCC of the primary constitutes the minority of the cases

13. Regarding minor salivary gland carcinoma


a. ( ) It occurs at hard palate
b. ( ) It is characterised by intact mucosa
c. ( ) Stage III is adequately treated surgically
d. ( ) Mucoepidermoid carcinoma is the second most common
e. ( ) Nodal involvement carries 60-70 percent 5 years survival
rate
NOV 2017
1) Parapharyngeal mass
a. ( ) Biopsy via intraorally
b. ( ) Causes medialization of lateral pharyngeal wall
c. ( ) CBT arises from medial part of carotid bulb
d. ( ) Vagal schwannoma causes splaying of ICA and ECA
e. ( ) Neck pain is due to irritation to pharyngeal plexus

2) H&N manifestations of AIDS


a. ( ) Oral candidiasis
b. ( ) Kaposi sarcoma
c. ( ) Malignant melanoma
d. ( ) Non Hodgkin lymphoma
e. ( ) Hairy leukoplakia of tongue

3) Complication of neck dissection


a. ( ) Causes winging of scapula
b. ( ) A/w pain on shoulder adduction
c. ( ) Causes subluxation of clavicle
d. ( ) Due to traction of SAN
e. ( ) Treat by cervical sympathectomy

4) Leukoplakia of oral cavity


a. ( ) Most common at buccal area
b. ( ) Seen mostly in alveolus
c. ( ) Treat with laser excision
d. ( ) 10% malignant transformation
e. ( ) Need life-long follow up

6) During selective supraomohyoid neck dissection


a. ( ) Anterior belly of digastric muscle is exposed
b. ( ) Marginal mandibular nerve is identified inferior to
facial artery
c. ( ) Submandibular gland is preserved
d. ( ) Lingual nerve is identified with retraction of stylohyoid
muscle
e. ( ) Hypoglossal nerve is preserved

7) Regarding pharyngeal pouch


a. ( ) Cause halitosis
b. ( ) CT scan is a good tool for assessment
c. ( ) RLN injury due to surgical repair
d. ( ) More common in left than right
e. ( ) Associated with malignancy

8) Surgery in OSA
a. ( ) 1 stline treatment in OSA
b. ( ) Mullers is a good investigation to determine the degree
of pharyngeal collapse
c. ( ) Surgery is adjunctive to CPAP
d. ( ) Friedmann classification is a useful predictor outcome
e. ( ) Lingual tongue retractor surgery, the base of tongue is
suspended to symphysis menti
9) Oral surgery + supraomohyoid neck dissection (indication)
a. ( ) Carcinoma in situ
b. ( ) Leukoplakia at buccal
c. ( ) Tongue SCC
d. ( ) Actinic keratosis
e. ( ) Spindle cell tumour of FOM

10) Cystic swelling over the angle of mandible


a. ( ) Hydratic cyst
b. ( ) Papillary cystadenoma lymphomatosum
c. ( ) HIV related cyst
d. ( ) Branchial cyst
e. ( ) Sjogren syndrome

11) Flap viability


a. ( ) Colour
b. ( ) Drain chart
c. ( ) Tissue turgor
d. ( ) Doppler
e. ( ) Dermal bleeding

12) Oncocytoma of salivary gland


a. ( ) 80% in MSG
b. ( ) Eosinophil granular cytoplasm
c. ( ) Enhanced Tc99 scintigraphy
d. ( ) Recurrent 50%
e. ( ) Early recurrent spread

13) Echelon drainage


a. ( ) Oral cavity: II, III and IV
b. ( ) Oropharynx: II, III and IV
c. ( ) Skin posterior to tragus: V
d. ( ) Subglottic: III
e. ( ) Hypopharynx: II, III and IV

14) Regional myocutaneous flap


a. ( ) PM flap
b. ( ) Latissimus dorsi flap
c. ( ) Deltopectoral flap
d. ( ) Temporal flap
e. ( ) Radial forearm flap

15) Fibrous dysplasia


a. ( ) Common in adult
b. ( ) Maxilla frequently affected
c. ( ) Pain common in extramaxillary
d. ( ) Increase in ALP suggest polycystic type
e. ( ) Interior to the bone causes of fibrous connective tissue

16) Dentigerous cyst


a. ( ) From infected tooth
b. ( ) In X-ray shows tooth with cyst
c. ( ) Usually asymptomatic
d. ( ) Commonly involve maxillary premolar
e. ( ) Originating from cell of Mallassez
NOV 2018
1. In parotid gland,
a. ( ) metastasis to the kidney is characteristic of adenoid cystic
carcinoma.
b. ( ) parotid lymphoma is associated with Sjogren Syndrome. .
c. ( ) the facial nerve trunk lies superficial to the retromandibular
vein.
d. ( ) the recurrence rate of mucoepidermoid carcinoma
correlates with its histological grade.
e. ( ) the secreto-motor fiber travels in the auriculotemporal
nerve.

2. Adenoid cystic carcinoma of the head and neck


a. ( ) arises from the minor salivary glands.
b. ( ) grows preferentially along the perineural sheaths.
c. ( ) high risk of occult neck metastasis in N0.
d. ( ) is slow growing.
e. ( ) radiotherapy is the treatment of choice.

3. Deep neck abscess


a. ( ) it includes Ludwig angina.
b. ( ) Klebsiella sp. is a common organism.
c. ( ) lateral neck x-ray is adequate to diagnose retropharyngeal
abscess.
d. ( ) parapharyngeal abscess is drained intraorally.
e. ( ) parotid abscess is drained via skin crease incision.

4. Oral cancer,
a. ( ) HPV-related SCC exhibit poor differentiation.
b. ( ) neck dissection is indicated in T 2 N 0 tumour.
c. ( ) tumor thickness has a significant correlation with occult
nodal disease.
d. ( ) ulcerative carry poorer prognosis.
e. ( ) verrucous carcinoma has the same prognosis with SCC.

5. Regarding head and neck sarcoma


a. ( ) It occurs in Li-Fraumeni syndrome.
b. ( ) It shows biphasic age distribution.
c. ( ) Lymph nodes metastasis is common.
d. ( ) Mainstay treatment is surgery.
e. ( ) Trojani grading is based on degree of necrosis.

6. Regarding osteoradionecrosis
a. ( ) Antibiotic is indicated.
b. ( ) Hyperbaric oxygen therapy is a treatment option.
c. ( ) It presents with non-healing ulcer with denuded bone.
d. ( ) Radiation arteritis is a feature.
e. ( ) Radio iodine ablation is a risk factor.

7. Medullary thyroid carcinoma,


a. ( ) arises from follicular cells.
b. ( ) is associated with Multiple Endocrine Neoplasia syndrome
Type I.
c. ( ) is monitored with serum calcitonin.
d. ( ) is treated with adjuvant radiotherapy.
e. ( ) presented with cervical lymphadenopathy in 90% of cases.
8. Regarding chyle leak
a. ( ) Daily output of 200 mls requires surgery.
b. ( ) High fat diet prior to exploration will increase lymphatic
flow.
c. ( ) Most commonly associated with level V dissection.
d. ( ) Octreotide is a treatment.
e. ( ) Requires long-chain triglycerides diet.

9. Regarding neck dissection for head and neck malignancy


a. ( ) Accessory nerve is preserved in radical neck dissection.
b. ( ) Level VI dissection is performed in Modified Radical Neck
Dissection.
c. ( ) Shoulder syndrome is caused by injury of the spinal
accessory nerve.
d. ( ) Simultaneous bilateral radical neck dissection is
contraindicated.
e. ( ) Untreatable primary tumor is a contraindication.

10. Regarding hypopharyngeal carcinoma


a. ( ) 80% of patients are N+ at time of presentation.
b. ( ) Benign tumours are common.
c. ( ) Post cricoid carcinoma is commoner in women.
d. ( ) Sites include pyriform fossa, post cricoid and vallecula.
e. ( ) T3 tumours presents with fixed hemilarynx.

11. Regarding lesions of the oral cavity


a. ( ) Behcet syndrome is characterized by ulcers of the genitalia.
b. ( ) Candidiasis appears as white area.
c. ( ) Carcinoma is commoner in upper lip.
d. ( ) Epulis is a premalignant lesion.
e. ( ) Leukoplakia can be rubbed off.

12. Thyroglossal duct cyst


a. ( ) Results from developmental abnormality of thyroid
descent.
b. ( ) Lump moves with swallowing.
c. ( ) Infected cyst should be excised immediately.
d. ( ) Removal of greater horn of hyoid is associated with low
recurrence rate.
e. ( ) More than 50% are suprahyoid.

13. Regarding benign jaw lesions


a. ( ) Ameloblastoma presents with multilocular radiolucent
lesion.
b. ( ) Aneurysmal bone has an epithelial lining.
c. ( ) Eruption cyst is a dentigerous cyst.
d. ( ) Fibrous dysplasia is commoner in the mandible compared
to the maxilla.
e. ( ) Nasopalatine cyst is an odontogenic cyst.
MAY 2019

1. AJCC:
a. ( ) Depth of invasion in oral CA added T
b. ( ) P16 pos poorer prognosis F
c. ( ) Parotid node 6cm = N3a
d. ( ) Vascular invasion determines prognosis

2. Flaps:
a. ( ) Blood supply to ALT- Descending branch of lateral femoral
circumflex artery T

NOV 2019
1. Carotid body tumour
a. ( ) Causes hypoglossal nerve palsy
b. ( ) Also known as chromaffin paraganglioma

2. Vagal paraganglioma
a. ( ) Neuroectodermal origin
b. ( ) Vagal causes horner’s syndrome
c. ( ) Involved nodus ganglion

3. Parotid SCC
a. ( ) Aggressive
b. ( ) Unencapsulated
c. ( ) Total parotidectomy
d. ( ) Facial nerve uncommonly affected
e. ( ) Commonly present with neck nodes

4. Benign salivary tumour


a. ( ) Warthins known as adenolyphoma T
b. ( ) Oncocytoma more common in submandibular gland
c. ( ) Pleomorphic absent of capsule

5. Malignant salivary gland tumour


a. ( ) Adenoid cystic commonest in submandibular gland
b. ( ) Acinic cell ca typical Perineural
c. ( ) Minor salivary gland tumour of soft palate best treated with
surgery
d. ( ) Ca Ex pleomorphic need neck dissection

6. Retropharyngeal abscess ?causes?


a. ( ) Sinusitis
b. ( ) Vertebral fracture

7. PSG
a. ( ) Home test is level 2
b. ( ) Level 3 can diagnose mild OSA
c. ( ) Split night is adequate to diagnose
d. ( ) First night reduces accuracy
8. OSA
a. ( ) Retropalatal collapse treated with pillar implants

9. Facial pain
a. ( ) Myofacial pain in post menopausal women
b. ( ) TMJ pain present at temporoparietal headache
c. ( ) Supraorbital artery compresses supratrochlear nerve,
Trigeminal neuralgia

10. Selective neck dissection


a. ( ) T1N1 tongue Ca
b. ( ) T3N0 Verrucous lip
c. ( ) T2N0 scalp ca
d. ( ) T3N0 low grade mucoepidermoid
e. ( ) T3N0 laryngeal Ca

11. Indications for selective neck dissection upper aerodigestive tract


carcinoma:
a) ( ) Tongue ca : T1n1m0- true
nak buat MRND pun x salah
b) ( ) Ca larynx: T3n0m0- true

c) ( ) Myoepithelial ca of parotid: t2n0m0- false
- low grade tumor.
d) ( ) Verrucous carcinoma of the lip t3n0m0- false
- verrucous is well differentiated
e) ( ) Scalp scc t2n0m0- false
N0 , no need neck dissection

12. Infectious mononucleosis
a. ( ) Treated with penicillin
b. ( ) Koplik spot
c. ( ) Caused by strep A
d. ( ) Hepatosplenomegaly
e. ( ) Cervical nodes
NOV 2020
1. N0 oropharyngeal ca
a. Gold standard for assessment is supraomohyoid neck dissection
b. Modified Blair is used for supraomohyoid neck dissection

2. Indications of adjuvant radiotherapy


a. Stage 3, 4
b. Single positive node
c. Extracapsular extension
d. Lymphovascular invasion
e. Perineural invasion

3. Neck abscess
a. Parapharyngeal space until C6
b. Retropharyngeal abscess treat with antibiotic
c. Salmonella sp common in diabetic
d. Same pathology in adult and children

4. Oral cavity lesion


a. Submucosal fibrosis due to HPV
b. 25% of leukoplakia will have malignant change
c. RT is best treatment in lateral tongue ca
d. Lichen planus presents with vesicobullous lesions
e. Geographical tongue need biopsy

5. Neck dissection
a. Spinal accessory nerve is part of consternation?
b. Supraomohyoid use modified blair incision

6. Papillary thyroid ca
a. More lymphatic extension compared to haematogenous
b. Ground glass appearance in histo
c. Treated with radiotherapy
d. Microcarcinoman if less than 1.5cm
e. Intratracheal invasion is treated with RAI

7. Parapharyngeal mass
a. Vagal paraganglioma push the carotid artery posterolaterally
b. Parapharyngeal mass can present as parotid lesion
c. Biopsy done after imaging
d. 70 % benign

8. Hypopharyngeal tumour
a. Radiotherapy is indicated for T1 tumour
b. Esophagectomy is indicated for pyriform fossa tumour

9. OSA
a. Radio-frequency of soft palate is indicated for Isolated retropalatal lesion
b. Bariatric surgery is indicated for BMI more than 40
c. Muller's maneuver poor in assessing the severity of obstruction
10. Recurrent salivary gland tumour
a. More in submandibular gland
b. More common in high grade tumour
c. Occurs after removal of parotid tumour
d. FNAC is indicated
e. Higher risk of facial nerve injury

11. Salivary gland tumour


a. Mucoepidermoid ca is the commonest salivary gland malignancy
b. 50% of submandibular gland tumour is benign
ADDITIONAL SETS : H&N
1. The following are true of parotid tumours
a. Adenoid cystic carcinoma is the commonest malignant tumour
in children
b. Non-Hodgkin’s lymphoma may require a superficial
parotidectomy to confirm the diagnosis
c. The incidence of salivary fistula after superficial parotidectomy
is higher in neoplastic cases compared to inflammatory cases
d. Distant metastases after many years are characteristic of
adenoid cystic carcinoma
e. Parotid facial nerve neuroma with normal facial movement
should be operated early

2. The following nerves are identified during a radical neck dissection


a. Hypoglossal nerve
b. Recurrent laryngeal nerve
c. Spinal accessory nerve
d. Phrenic nerve
e. Lingual nerve

3. The following are true of salivary glands


a. Submandibular duct has an intimate relationship with the
hypoglossal nerve
b. The lingual nerve lies lateral to the submandibular duct at its
opening in the floor of the mouth
c. The auriculotemporal nerve carries secretomotor nerve fibres to
the parotid gland
d. To avoid injury to lingual nerve, the submandibular duct is best
approached surgically through the mouth
e. The superior salivary nucleus supplies preganglionic
parasympathetic nerve fibres to the submandibular gland

4. The following are true in reconstruction in head and neck surgery


a. The nasolabial flap is useful in repairing defects in the anterior
part of the floor of the mouth
b. The nasolabial flap is based on its blood supply from the inferior
labial artery
c. The split skin graft is useful in covering defects of the mandible
d. Full thickness grafts are commonly used on granulating surfaces
to cover the defects

5. Regarding hypopharyngeal cancer


a. Pyriform sinus cancer is most common
b. Hoarseness is the commonest symptom
c. Barium swallow is a useful investigation to confirm the diagnosis
d. Plummer-Vinson syndrome is associated with post- cricoid
carcinoma
e. Pyriform sinus carcinomas metastasizes late to the neck nodes

HEAD AND NECK


1 FTTTF
2 TFTFT
3 FFTFT
4 T F F F
5 TFFTF
USM COLLECTION
1. In hypopharyngeal carcinoma
a. The postcricoid is the most common site
b. It is associated wit Paterson-Kelly syndrome
c. Laryngeal crepitus is lost in postcricoid tumour
d. The high incidence of ‘skip lesion’ is due to dissemination of
tumours in submucosal lymphatic
e. The post cricoid tumours have a tendency to spread to
paratracheal nodes
FTTTT

2. With regards to complication of thyroidectomy


a. Unexplained occipital headache is due to injury to the greater
auricular nerve during division of the strap muscles
b. Circumoral paraesthesia is due to hypothyroidism
c. Chvostek’s sign is elicited by twitching of facial muscle following
tapping parotid region
d. Tracheomalacia may developed secondary to injury of trachea
e. Trousseau’s sign is elicited by occlusion of the radial artery
FFTFF

3. In benign thyroid disease


a. Subacute thyroiditis is secondary to an acute viral infection
b. There is risk of developing thyroid lymphoma in Graves’s disease
c. Dysthyroid eye disease is caused by exophthalmos-producing
substances
d. Hard-rock thyroid mass suggest Hashimoto’s thyroiditis
e. Acute thyroiditis is often due to an acute granulomatous
infection
TFTFT

4. The following are true


a. A cold nodule in thyroid scan is rarely malignant
b. Compression of the neck mass to trachea is better viewed in
lateral neck X-ray as compared to AP view
c. In long standing goitre, sudden increase in size and pain
indicates malignant transformation
d. Papillary thyroid carcinoma is the most common type of
malignant transformation in long standing goitre
e. The external jugular vein is a good landmark for elevation of
subplastymal flap in thyroidectomy
FTTTF

5. The following are true


a. Basal cell carcinoma is more common in the upper lip than the
lower lip
b. The most common site for oral cancer is at the lateral border of
tongue
c. The upper lip has bilateral and ipsilateral lymphatic drainage into
level I-III nodes
d. In lip cancer poorer prognosis is associated with involvement of
the commissure
e. Oral cavity consists of buccal mucosa, upper and lower alveoli,
hard and soft palate, anterior two-thirds of the tongue , floor of
mouth and the anterior tonsillar pillars
TTFTF
6. In hypopharyngeal tumour
a. The most common non-squamous malignancies is adenoid cystic
carcinoma
b. Haemoptysis may occurs in post cricoid tumour
c. Unilateral pooling of saliva indicate piriform fossa tumour
d. In clinically N0 neck, the use of ultrasound guided fine needle
aspiration is superior to CT Scan in confirming neck node
involvement
e. Peristomal recurrence may occur when the paratracheal node is
not cleared surgically
TFTTT

7. In oral cavity carcinoma


a. 90% of malignant tumour are SCC
b. Tumours involving the alveolus tend to spread to submandibular
gland
c. Tumours of the anterior -two -third of tongue may spread
directly to jugulo-omohyoid nodes
d. Second primary tumours can occur in lungs
e. Involvement of intrinsic muscles of the tongue alone is classified
T4
TFTTF

8. Which of the following statement(s) is/are true


a. Erythroplakia appears as a white patch in the buccal mucosa
b. Eryhtroplakia is a premalignant condition in the oral cavity
c. Erythroplakia represent dysplasia with keratosis
d. The reticular type of lichen planus give rise to erythematous
lesion of the buccal mucosa
e. The histological changes in leukoplakia include acanthosis
FTFFT

9. In the cancer of lip


a. Actinic cheilitis is a premalignant condition
b. Lesion in the upper lip has better prognosis than in the lower lip
c. The mucosal lining of the lip is squamous epithelium
d. Exposure to sun is a major predisposing factor
e. The upper lip is more commonly affected than the lower lip
TFTTF

10. In the reconstruction of lip surgery


a. Abbe Estlander flap can be used to repair laterally placed tumour
of upper lip
b. Drooling fissuring of the tissues are common disadvantages in
Karapandzic principle
c. Reverse Karapandzic flap is useful for reconstruction of upper lip
lesion
d. For SCC measuring less than 1 cm, the accepted margin
clearance is 5mm
e. Orbicularis oris muscle should not be stitched during lip repair
FFTTF
11. Anatomy of the lip
a. It is included in the oral cavity
b. Mucosa contains accessory salivary tissue
c. The labial arteries lie in the submucosa between the mucosa and
orbicularis oris muscle
d. Lymphatic drainage of the upper lip includes the pre parotid
lymph nodes
e. Lymphatic drainage of the lower lip includes the jugulodigastric
nodes
TTTTF

12. In cystic hygroma


a. The commonest site is at the anterior triangle of neck
b. There are commonly ill-defined anatomical planes
c. The swelling is compressible
d. The cyst can be unilocular
e. Cholesterol crystal is one of the histological features
FTTTT

13. Regarding neck metastasis


a. Tumours of the lateral part of the lower lip tend to metastasize
to jugulodigastric nodes
b. Midline lesions of the upper lip tend to metastasize to submental
nodes
c. The five year survival rates falls from 95% to 80% when positive
nodes are present
d. The involvement of jugulo-omohyoid nodes can occur in lesion
anteriorly placed In floor of mouth
e. Contralateral nodes are more likely in lesions involving dorsum
of tongue
FFFTT
RHINOLOGY

NOV 2007
1. Angiofibroma
a. Bowing posterior wall of maxillary sinus
b. Tend to break the capsule and grow
c. Fibroangioma more aggressive than angiofibroma
d. Can involve the intra-orbital structure by premature pathology
e. Arise from sphenopalatine junction
TFFT
2. Inverted papilloma
a. Can effect maxillary and ethmoid sinuses
b. Common at the nose floor
c. Total maxillectomy is the treatment of choice
d. It eroded the bone
TFFF

3. Sinus and para sinus tumour


a. Squamous epithelium is common tumour in the area
b. Optic chiasma is posterior-inferior to the sphenoid sinus
c. In the sphenoid sinus involve of postero-superior aspect the
cavernous sinus
d. Adeno ca is related to hard wood worker

TFTT

4. The drainage of anterior group of paranasal sinus


a. Maxillary ostium
b. Anterior ethmoid
c. Ethmoid infundibulum
d. Frontal sinus
TTTT

5. Wegener’s granulomatous disease


a. C- ANCA diagnosed test
b. Chemotherapy is drug of choice
TF
MAY 2008
1. Olfaction
a. Cerebral input is received by trigeminal nerve
b. Total olfaction loss lead to complete anosmia
c. Viral cause of olfactory disease is permanent
d. The development complete by 4 years old
TFFF

2. Intrinsic rhinitis
a. The most common symptoms is nasal congestion and anosmia
b. More to occur in children than adult
c. Non eosinophilic – response to intranasal corticosteroid
d. Skin prick test invalid in patient on antihistamine
e. Polyp in non eosinophilic
TFFTF

3. In food allergy
a. Egg yolk is the commonest food allergy
b. Allergic reaction develop within 1 hr of ingestion
c. Intradermal adrenaline in adult is 0.3mg
d. Type IV, most common in nickel and cobalt
FTFT

4. In AFS
a. RAST is positive
b. Histology showed abundant of Charcot Leyden
c. Allergic mucin shows fungal hyphae
d. In the mucosa there is fungal invasion
TTTF

5. In skin testing
a. Subcutaneous adrenaline is needed
b. Patient should be explained regarding systemic side effect
c. Beta blocker should be avoid before
d. Should be done in patient in intact immune system
TTFF

6. In Wegner granulomatosis
a. Septal perforation one of presenting symptom
b. Pathology same as polyartheritis nodosa
c. 95% increase ANCA in acute phase
d. Radiotherapy for recurrent case
e. Cisplatin Is the treatment of choice
TTTFF

7. PNS tumour
a. Neck node is earliest manifestation
b. SCC is commonest
c. Ameloblastoma is benign and fast-growing tumour
d. Adenoid cystic CA common in wood worker in furniture
industries
e. Burkitt’s lymphoma is confined in African children
FTFFF
8. Anatomy of PNS
a. Extensive anterior middle meatal antrostomy will result in
epiphora
b. The shallow depression anterior to middle turbinate is agger nasi
c. Hiatus semilunaris is below the bulla ethmoidalis
d. Anterior end of hiatus semilunaris is the infundibulum
e. Bulla ethmoidalis is develop from anterior ethmoidal cells
TFFFT

9. Inverted papilloma
a. Bony involvement is due to tumour infiltration
b. Concurrent malignancy is commoner than malignant
transformation
c. HPV 16 causes malignant transformation
FTF

10. Fracture of nasal bone


a. Chevallet type involved the nasal septum
b. Type 2 involved PPE
c. Telecanthus oocur in nasoethmoidal fracture
d. Type 3 need open reduction external reduction
e. Jarjavay fracture nasal septum a/w anterior force
TTTTT

11. CSF rhinorrhea


a. Is demonstrated by asking patient to lean forward and do
Valsalva
b. Mucoprotein Is abundant
c. The discharge is viscous
TFF
12. Complication od ESS
a. Orbital haematoma rarely cause by trauma to lamina papyracea
b. It causes post op scarring
c. CSF leak
d. Medial rectus palsy
e. Lacrimal duct injury
FFTTT
NOV 2008
1. Regarding the mycetoma form of fungal sinusitis
a. Causative organism has the ability to invade the sinus mucosa
b. Bony erosion of the sinus wall is an early feature
c. Patients on long term antibiotics are especially proned
d. Patient are immuno-competent
e. CT scan shows areas of radio-opacity that have a similar density
to bone
FFTTT

2. Regarding inverted papilloma


a. It consists of epidermoid cells
b. The basement membrane remains intact
c. Papilloma containing HPV 16 have a greater tendency to
undergo malignant transformation
d. Co-existent malignancy is commoner than malignant
transformation
e. Current practise is five years post operative follow up
TTTTT

3. Rhinocerebral mucormycosis
a. Is an opportunities disease
b. Histopathologically is characterised by extensive thrombosis
c. Is recognized in tissue sections by uniform acute angle branches
of the hyphae
d. Shows predominantly polymorphonuclear cells
e. Sphenoid sinus remains disease free until the end
TTFTF

4. Regarding the aspirin intolerance , chronic sinusitis and asthma triad


a. It apply to all non-steroidal anti inflammatory drugs
b. It affects a higher proportion of women
c. It manifests before the age of 30 years
d. Skin sensitivity tests are positive for conventional allergens
e. Intolerance develops after a preliminary phase of eosinophilic
rhinitis
TTFFT
5. The following statement are true regarding turbinate hypertrophy
a. It is bony in origin
b. Nasal CPAP causes inflammatory hypertrophy of inferior
turbinates
c. Medical therapy is the first line of treatment
d. Inflammation secondary to non-allergic rhinitis is an important
cause
e. A symptomatic mulberry posterior tip not responding to medical
therapy should be excised
TFTTT

6. The following statement concerning skin prick test re true


a. Patient must have an intact immune system
b. Patient must be free of skin disease
c. Contraindicated in unstable cardiovascular disease
d. Beta-blockers should be avoided prior to skin testing
e. Subcutaneous adrenaline must be available
FTTTT
7. The following statements are true of hereditary haemorrhagic
telangiectasia
a. It is an autosomal dominant disease
b. Androgens are used in treatment
c. Injection of sclerosant is curative
d. The mode of treatment involves septodermoplasty
e. It is associated with bleeding from the gut
TFFTT

8. Septal hematoma
a. Usually traumatic in origin
b. May be due to blood dyscrasia
c. Unilateral nasal obstruction is the commonest symptom
d. They are like to resolve spontaneously without complication
e. Treatment is conservative
TTFFF

9. Nasal lupus vulgaris


a. This is tuberculous infection of the vestibule
b. It is related to open exposure to tuberculosis
c. Histologically shows ‘apple jelly’ nodules
d. The bony septum may perforate
e. Antituberculosis prevents deformity
TTFFF

10. Regarding antrochoanal polyps


a. Are commonest in elderly
b. Are multiple
c. Associated with rhinorrhoea
d. Opacity Is seen in maxillary sinus x-ray
e. Associated with allergy
FFFTF

11. With regards to malignant tumours of the nose and paranasal sinuses
a. Adenocarcinoma is the commonest type
b. Lymph node metastases are rare
c. An ameloblastoma is rapidly growing
d. Ringertz tumour is part of them
e. Olfactory neuroblastoma should be ruled out if undifferentiated
cells are seen histologically
FTFFT

12. In food allergy


a. Egg yolk is a common allergen for IgE mediated hypersensitivity
b. Allergy reaction develop within first hour of ingestion
c. Most children lose their sensitivity withing 5 years of life
d. Atopic dermatitis is non IgE mediated allergic response
e. The recommended intramuscular adrenaline adult dose is 0.3mg
FTTTT
MAY 2012

1. Embryological development of the nose


a. ( ) Maxillary sinus develop at 4 thweek IUL
b. ( ) Sphenoid sinus is fully developed by 12 yo

2. Acoustic rhinometry
a. ( ) Measures circumferential area using USS waves
b. ( ) Measures via both anterior and pposterior method

3. Fungal sinusitis
a. ( ) 90% is caused by aspergillus fumigates
b. ( ) Fulminant allergic fungal sinusistis occurs in
immunocompetent patients
c. ( ) Aspergilloma : presence of Charcot Layden crystal

4. Nasal polyp
a. ( ) Commonest cause is fungal sinusitis
b. ( ) Is due to poor lymphatic drainage

MAY 2013
1. Inverted papilloma
a. ( ) Treated by complete surgical excision
b. ( ) Less than 1 % malignant change
c. ( ) Also known as transitional cell papilloma
d. ( ) Occurs commonly in septum

2. Samters triad
a. ( ) Irreversible
b. ( ) Affect cyclo-oxygenase pathway
c. ( ) Increased leukotrienes
d. ( ) Aspirin sensitivity is uncommon
e. ( ) Associated with bronchiectasis / nasal polyposis/ aspirin

3. Frontal mucocele
a. ( ) Frontal is the commonest type in PNS
b. ( ) Type 1…..
c. ( ) Type 2 erode orbit
d. ( ) Endoscopic decompression is the treatment of choice

4. Churg Strauss syndrome


a. ( ) Eosinopenia is characteristic
b. ( ) Also known as allergic granulomatous angiitis
c. ( ) Treatment with glucocorticoid alone is adequate
d. ( ) ESR raised
e. ( ) Is associated with bronchial asthma

5. Olfactory
a. ( ) Ammonia is used to test for olfaction
b. ( ) Infants can smell ?something about olfaction is not
matured at birth
6. Chordoma
a. ( ) 80% occurs in the region of skull base
b. ( ) It arise from vestigial notochord remnant
c. ( ) Preoperative RT is given to reduce tumour bulk
d. ( ) Deafness is one of the presenting features
e. It causes extensive bone destruction

7. Cystic fibrosis
a. ( ) Autosomal dominant
b. ( ) Male predominant
c. ( ) The incidence of nasal polyp decrease with age
d. ( ) Meconium ileus is a common presenting symptom
e. ( ) The sinuses act as a reservoir for descending infection

8. Wegener granulomatosis
a. ( ) Autosomal recessive
b. ( ) cANCA correlates with disease activity
c. ( ) absence of renal involvement is associated with 5 years
survival rate 100%
d. ( ) affect small vessel
e. ( ) Bactrim can reduce disease relapse

9. Radiotherapy SE in NPC
a. ( ) Dental caries
b. ( ) Hyperpituitarism
c. ( ) Xerostomia
d. ( ) Hypothyroidism
e. ( ) Avascular necrosis

10. CSF leak


a. ( ) Increase flow by jugular compression
b. ( ) Defect in middle cranial fossa goes into ethmoid sinus
c. ( ) Spontaneous leak seen in ppl of 20 years old
d. ( ) Low pressure leak headache relieved by straining

11. Midline T- cell lymphoma


a. ( ) Radical radiotherapy treatment of choice
b. ( ) Abundant atypical lymphocyte
c. ( ) Granuloma (Giant cell are seen histologically)
d. ( ) No renal involvement

15. Rhinosinusitis with orbital complication


a. ( ) More in young adult
b. ( ) Color blindness first sign
c. ( ) Orbital cellulitis show proptosis
d. ( ) In orbital apex syndrome, ophthalmoplegia is permanent
e. ( ) In orbital abscess , there is permanent ophthalmoplegia

16. Nasal resistance


a. ( ) Nasal resistance is minimum in infant compared to adults
b. ( ) Contributes 2/3 of upper airway resistant
c. ( ) Highest resistant between septum and turbinate
d. ( ) The nasal valve is situated just anterior to the tip of the
inferior turbinate
e. ( ) Sympathetic stimulation increases nasal resistance
17. Regarding sinonasal malignancy
a. ( ) 70% is SCC
b. ( ) Involve pterygoid plate is T4b
c. ( ) 70% of sinonasal ca arises from the maxilla
d. ( ) In ca ethmoid , involvement of the maxilla is stage at T2
e. ..

18. Regarding nasal malignancy


a. ( ) Columellar SCC Is aggressive and Is associated with bad
prognosis
b. ( ) Vestibule SCC affect young people
c. ( ) T1 N1 is stage 3

19. True regarding SPT


a. ( ) It is indicated in patients with eczema
b. ( ) Negative SPT does not exclude allergic disease
c. ( ) SPT is superior than intradermal skin test
d. ( ) Dermatographism occurs in hypersensitive patients
e. ( ) Demonstrate Ig E mediated sensitization to inhalant/ food
allergens
MAY 2014
1. Nasopharygeal carcinoma
a. ( ) N3 is stage IVa
b. ( ) Undifferentiated type Is associated with EBV
c. ( ) MRI is superior to CT scan in assessment for recurrence

2. Fungal sinusitis
a. ( ) Candida is the commonest pathogen
b. ( ) Acute invasive fungal sinusitis need surgery
c. ( ) During endoscopic surgery will show ‘brown sludge’
d. ( ) Silver stain used for fungal stain
e. ( ) Invasive fungal sinusitis will cause proptosis

3. Inverted papilloma
a. ( ) Is more common in male
b. ( ) Malignant potential in 10%
c. ( ) HPV type 11 is associated with the malignant change
d. ( ) Involvement ethmoid is FISCH type 2

4. Modified Lothrop
a. ( ) Preserve the frontal process of maxilla
b. ( ) Prevent medial collapse of orbit
c. ( ) Is a frontal drill-out procedure
d. ( ) Preserve the superior nasal septum
e. ( ) To increase the frontal sinus drainage

5. Juvenile nasopharyngeal angiofibroma


a. ( ) Embolization is mandatory
b. ( ) Stage 3c is intradural
c. ( ) Biopsy is indicated

6. Regarding olfaction
a. ( ) Glossopharyngeal nerve has contribution in olfaction
b. ( ) Viral infection can cause permanent loss of olfaction
c. ( ) Ammonia is a reliable test material
d. ( ) Alzheimers causing olfaction problem

7. True regarding sinusitis


a. ( ) Subacute rhinosinusitis occur more than 12 weeks
b. ( ) Viral rhinosisnusitis will resolve within 10 days
MAY 2015
1. Paranasal sinus development
a. ( ) Frontal sinus is radiologically seen at 12 month
b. ( ) Maxillary sinus first to develop
c. ( ) The superior lateral indentation at maxillary sinus is the
maxillary nerve
d. ( ) at birth, anterior ethmoid cells are big enough to have
clinical significant
e. ( ) Paransal sinus get infected when OMC is blocked

2. Pott’s puffy tumour


a. ( ) Complication of acute sinusitis
b. ( ) Cracking sound on palpation
c. ( ) Bony erosion
d. ( ) Treatment of choice is surgery
e. ( ) Eye are pushed inferomedially

3. SNUC ? differentiated sinonasal tumour


a. ( ) Due to nickel
b. ( ) Due to sun exposure
c. ( ) Low incidenc of neck mets
d. ( ) Commonly arise from lateral nasal wall
e. ( ) Treat like NPC

4. Septal perforation is associated with


a. ( ) TB
b. ( ) Wegener’s granulomatosis disease
c. ( ) Leprosy
d. ( ) Syphilis
e. ( ) Lymphoma
f. ( ) Eosinophilic non allergic rhinitis

5. Regarding nasal packing


a. ( ) Need to cover with systemic antibiotics
b. ( ) Complicated by toxic shock syndrome
c. ( ) SE concern meningoencephalitis
d. ( ) Contraindicated in skull base #
e. ( ) Cocaine will cause myocardial infarction

6. Skin prick test


a. ( ) In vivo test
b. ( ) Antileukotriene cause false negative
c. ( ) Need to stop antihistamine 4 weeks
d. ( ) Dermatographism is contraindicated
e. ( ) If bleeding , high risk of systemic complication

7. Naso orbital fracture


a. ( ) Fracture or pterygoid plate in Le Fort ..
b. ( ) Close reduction is treatment in ….

8. Regarding craniofacial Actinomycosis


a. ( ) The invaded Adjacent tissue become soft
b. ( ) Sulfur granule
c. ( ) Actinomycosis israelli is aerobic bacteria
d. ( ) Severe Periodontis is a feature
e. ( ) Suppurative pneumonia Is a complication
9. T-cell lymphoma of nose
a. ( ) Giant cell
b. ( ) Atypical lymphocyte is found in submucosa
c. ( ) No renal involvement
d. ( ) Radical radiotherapy is a treatment of choice
e. ( ) Unhealed ulcer
f. ( ) Non Healing midline granuloma

10. Granulomatous infection of the nose


a. ( ) Secondary syphilis present as rhinitis
b. ( ) DAPSON is treatment for leprosy
c. ( ) Miculicz cell is associated with rhinoscleroma
d. ( ) Nasal TB secondary from TB pulmonary
e. ( ) Vestibulitis with cavernous sinus thrombosis mortality rate
is 60%

11. Bilateral grade III nasal polyp


a. ( ) Optic neuropathy?
b. ( ) Anosmia
c. ( ) Mucocele
d. ( ) Proptosis
e. ( ) Tropical eosinophilia

12. Regarding nasal resistance


a. ( ) Nose Contribute 2/3 of airway resistance
b. ( ) Turbinate is biggest contribution
c. ( ) Is the main airway resistant in infant
d. ( ) Exercise reduces nasal resistance
e. ( ) Nasal valve is part of it

13. Nasal glioma


a. ( ) 5% from all congenital nasal mass
b. ( ) Compressible
c. ( ) a/w CN deficient about 60%
d. ( ) Treated with Radiotherapy
e. ( ) Size Increase in straining
NOV 2015
1. Blow out fracture
a. ( ) Ocular movement limited
b. ( ) Cause by blunt injury
c. ( ) Fracture infraorbital rim
d. ( ) Treat by Caldwell Luc
e. ( ) Tear drop sign

2. Allergic rhinitis
a. ( ) Intradermal kin test more sensitive than skin prick test
b. ( ) Serum Ig E higher than normal population
c. ( ) RAST measures allergen specific Ig E
d. ( ) Sodium cromoglycate is prophylactic agent
e. ( ) Long term macrolide has a role

3. Minor symptom of rhinosinusitis


a. ( ) Headache
b. ( ) Nasal block
c. ( ) Halitosis
d. ( ) Fever
e. ( ) Anosmia

4. NPC
a. ( ) Hematogenous spread more frequent
b. ( ) Age commonly affected 51 to 60
c. ( ) Male to female ratio 4-5:1
d. ( ) Associated with EBV
e. ( ) Type 1 more common

5. Olfactory neuroblastoma
a. ( ) Origin from neural crest
b. ( ) Young age more metastasis
c. ( ) Old age recurrence
d. ( ) Slow growing
e. ( ) Main symptom is epistaxis

6. Angiofibroma
a. ( ) Non aggressive
b. ( ) From posterior nasal space
c. ( ) Common presented with facial swelling
d. ( ) Mainly fibrous element in older patient
e. ( ) Contain androgen receptor

7. Hereditary Haemorrhagic Telangiectasia (Osler Weber Rendu syndrome)


a. ( ) Androgen is treatment
b. ( ) Severe epistaxis
c. ( ) Associated with pulmonary AVM
d. ( ) Septal dermoplasty
e. ( ) Treatment is full thickness skin graft

8. AFRS (allergic fungal rhinosinusitis)


a. ( ) An immunocompetent
b. ( ) Atopy detected in AFRS
c. ( ) Rhizomucor in invasive
d. ( ) Granulomatous chronic invasive in immunocompromised
e. ( ) Orbital apex syndrome in chronic invasive fungal sinusitis
9. CSF Leak
a. ( ) Beta trace protein to confirm CSF
b. ( ) Amount of leakage profuse in non traumatic leak
c. ( ) Complication of intrathecal fluorescent involved grand mal
seizure
d. ( ) Surgical intervention indicated if conservative failed more
than 6 month

10. Acoustic rhinometry


a. ( ) Nasal cycle effect the result
b. ( ) Based on sound reflected from nasal cavity
c. ( ) Useful in anterior part of nose

11. Barotrauma of PNS


a. ( ) Caused by rapid change in ambient pressure
b. ( ) Decongestant can relief symptoms
c. ( ) Balloon sinuplasty is a treatment
d. ( ) Severe pain during ascend
e. ( ) Epistaxis due to submucosal haemorrhage

12. Regarding orbital hematoma post FESS


a. ( ) Slow progressive proptosis require urgent decompression
b. ( ) Orbital dissection done to stop bleeding
c. ( ) Immediate referral to ophthalmology
d. ( ) Topical timolol reduce IOP

13. Sjogren syndrome


a. ( ) Steroid is treatment
b. ( ) Symptom is iritis
c. ( ) Risk of lymphoma
d. ( ) HLA type 1
e. ( ) Confirm by blood test
MAY 2016
1. Complication of midfacial degloving
a. ( ) Nasal Vestibular necrosis
b. ( ) Diplopia
c. ( ) Epiphora
d. ( ) Septal perforation
e. ( ) Oro-antral fistula

2. Le Fort I fracture
a. ( ) Causes of trismus
b. ( ) Causes of epistaxis
c. ( ) Fracture line crosses the maxillary sinus
d. ( ) Causes telecanthus
e. ( ) Fracture line is near in the infraorbital foramen

3. CSF leak
a. ( ) Intraoperative repair directly
b. ( ) Post op leak managed conservatively
c. ( ) MRI for detection ??
d. ( ) B2 transferin can detect

4. Potts Puffy tumour


a. ( ) Is associated with bone erosion
b. ( ) Is a complication of acute sinusitis
c. ( ) Has an eggshell crackling sensation on palpation
d. ( ) Causes inferior displacement of the orbit
e. ( ) Requires surgical excision

5. The following should be treated with allergen avoidance


a. ( ) Atrophic rhinitis
b. ( ) Gustatory rhinitis
c. ( ) Vasomotor rhinitis
d. ( ) Condensation rhinitis
e. ( ) Stress induced rhinitis

6. The following is true about orbital complication of rhinosinusitis


a. ( ) Proptosis occurs with orbital cellulitis
b. ( ) Preseptal cellulitis should be treated surgically
c. ( ) Typically occurs in younger patients
d. ( ) Orbital apex syndrome is associated with permanent
opthalmoplegia
e. ( ) Colour vision is affected first

7. JNA
a. ( ) Commonest benign nasopharyngeal tumour
b. ( ) Present in adult male
c. ( ) Radiotherapy indicated for infratemporal fossa extension
d. ( ) spine instrumentation is a cause

8. AFRS
a. ( ) RAST positive
b. ( ) Charcot Leyden crystal in allergic mucin
c. ( ) Biopsy shows invasion of sinus mucosa
d. ( ) SPT positive for fungal allergen
9. Orbital complication
a. ( ) Preseptal cellulitis treated by antibiotic
b. ( ) Orbital cellulitis cause proptosis

10. Olfactory neuroblastoma


a. ( ) Bimodal age
b. ( ) Metastasis in 10%
c. ( ) Recurrence in 80%
d. ( ) Homer- Wright rosettes is pathognomonic
NOV 2016
1. Regarding Potts Puffy tumour
a. ( ) Posterior table frontal sinus involved
b. ( ) Clinically present with egg shell crackling
c. ( ) It is associated with displacement of eyeball
d. ( ) A complication of acute sinusitis
e. ( ) Rx is with 1week antibiotic

2. Midfacial degloving complications are


a. ( ) Oroantral fistula
b. ( ) Septal perforation
c. ( ) Diplopia
d. ( ) Epiphora
e. ( ) Nasal vestibular necrosis

3. In olfactory neuroblastoma
a. ( ) Arise from stem cell of neural crest
b. ( ) Demonstrates Homer Wright rosette appearance on
histology
c. ( ) Shows more local recurrence in young patients
d. ( ) Presence primarily with parosmia
e. ( ) Stained positive for chromogranin

4. Basal encephalocele
a. ( ) Arises between cribiform plate and superior orbital fissure
b. ( ) Presents with hypertelorism
c. ( ) Presents as mass over the glabellar region
d. ( ) Expands while crying
e. ( ) Is surgically treated after puberty

5. Regarding mucoceles of sinuses


a. ( ) Sphenoid is the most commonly affected
b. ( ) Bone resorbing cytokines are absent
c. ( ) Cystic degeneration of glandular tissue is the cause
d. ( ) The clinical presentation is immediate
e. ( ) It is pus filled cavity

6. Granulomatous disease of the nose


a. ( ) Nodular type of nasal tuberculosis usually begins in the
nasal vestibule
b. ( ) Gumma syphilis of the nose is treated with intravenous
penicillin
c. ( ) Tuberculous leprosy involve nasal mucosa
d. ( ) Negative C-ANCA test excludes the diagnosis of Wegener
Granulomatous
e. ( ) Nasal involvement of sarcoidosis indicates multisystem
involvement of the disease

7. Regarding septal reconstruction during septoplasty


a. ( ) Rib cartilage is the first choice
b. ( ) Strip of cartilage is placed under the cartilaginous dorsum
c. ( ) Mattress suture is contraindicated
d. ( ) Nasal splint is effective
e. ( ) Nasal packing is maintained for 1 week
8. Predisposing factor for Eosinophilic predominant chronic sinusitis
includes
a. ( ) Atrophic rhinitis
b. ( ) Cystic fibrosis
c. ( ) Aspirin sensitive nasal polyps
d. ( ) Immune deficiency
e. ( ) Allergic rhinitis

9. Regarding fungal sinusitis


a. ( ) It is prevalent in arid condition
b. ( ) Angioinvasion occurs in mycetoma
c. ( ) The most common cause is aspergillus niger
d. ( ) Intravenous amphotericin is indicated in rhinocerebral
mucormycosis
e. ( ) Atopy is associated with the invasive type

10. Regarding haemangiopericytoma


a. ( ) It is capillary in origin
b. ( ) It arises from pericyte of Zimmerman
c. ( ) It has low recurrence rate
d. ( ) Surgery is the primary treatment modality
e. ( ) Preoperative embolization for large tumours

12. Regarding nasal dermoid cysts


a. ( ) It contains skin appendages
b. ( ) Is due to an embryonic fault between the developing nasal
bones and cartilaginous capsule
c. ( ) Present with superficial small lesion
d. ( ) Occur with associated dermal sinus
e. ( ) Encephalocele is a differential diagnosis

13. Topical intranasal steroids


a. ( ) Causes adrenal suppression over a long period
b. ( ) Causes nasal candidiasis
c. ( ) Causes epistaxis
d. ( ) Reduces nasal resistance in allergic rhinitis
e. ( ) Are effective at controlling nasal polyposis in cystic fibrosis

14. Regarding nasal furunculosis


a. ( ) It is an infection of a pilosebaceous follicle in the vestibule
b. ( ) Painful fissures occur in chronic vestibulitis
c. ( ) Commonest organism is staphylococcus aureus
d. ( ) Ciprofloxacin is the treatment of choice
e. ( ) Intraorbital complication is common

15. Structures that are encountered in frontal sinusotomy include


a. ( ) Agger nasi cell
b. ( ) Interfrontal sinus septl cell
c. ( ) Recessuss terminalis
d. ( ) Uncinate process
e. ( ) Suprabullar cell

16. Regarding orbital floor fractures


a. ( ) Increased intraorbital pressure causing thin bones of orbit
to ‘blow- out’
b. ( ) X-ray sinuses shows tear drop sign
c. ( ) Findings include subcutaneous emphysema
d. ( ) Enopthalmos is and early presentation
e. ( ) Forced-duction test is performed to test the extra ocular
mucles movement
NOV 2017
1) Wegener granulomatosis
a. ( ) Septal perforation one of the presenting symptom
b. ( ) Pathology same as PAN
c. ( ) 95% increase in ANCA in acute phase
d. ( ) Radiotherapy for recurrent use
e. ( ) Cisplatinum is the treatment of choice

2) Food induced rhinitis


a. ( ) Shellfish causes immediate symptoms
b. ( ) Type 1 and 3 immune response
c. ( ) Symptoms involved 2 or more organ
d. ( ) Present in 4 thdecade
e. ( ) A/w atopy

3) Inverted papilloma
a. ( ) Arise from the floor of nose
b. ( ) Total maxillectomy is the treatment
c. ( ) Erosion of bone is seen

4) JNA
a. ( ) Bowing of posterior wall of maxillary sinus
b. ( ) Direct extension to orbit via preformed pathology
c. ( ) Macroscopically presented as sessile polypoidal mass
d. ( ) Arise from pterygomaxillary fissure
e. ( ) Present with nasal obstruction

5) Allergic fungal sinusitis


a. ( ) Topical steroid
b. ( ) SPT is one of the management
c. ( ) Total IgE is raised

6) Granulomatous lesion of nose


a. ( ) Perforation in TB affects the cartilaginous part
b. ( ) Gumma in syphilis treated with IV penicillin

7) Absolute contraindication of SPT


a. ( ) Pregnancy
b. ( ) Infant
c. ( ) Severe dermatographism
d. ( ) Severe asthma
e. ( ) on Beta blocker

8) Olfactory neuroblastoma
a. ( ) MRI better than Ct for extension
b. ( ) Craniofacial resection followed by PORT is treatment
of choice
c. ( ) Mass in superior part of nasal cavity
9) Indication to explore frontal sinus in trauma
a. ( ) Posterior table fracture
b. ( ) Anterior table non displaced fracture
c. ( ) Persistent CSF rhinorrhea
d. ( ) Persistent unilateral opacity
e. ( ) Frontal recess disrupted
NOV 2018
1. Sniffin’ stick test,
a. ( ) comprises two tests.
b. ( ) discriminates odourants in triplets.
c. ( ) is an ampoule-like odour dispensing device.
d. ( ) tests the nasal chemosensory function.
e. ( ) uses N-butanol to test odour threshold.

2. Complications of rhinosinusitis,
a. ( ) CT scan is mandatory in all cases of suspected complication
of rhinosinusitis.
b. ( ) Endoscopic approach is favourable in the drainage of
laterally located orbital subperioesteal abscess.
c. ( ) Intravenous antibiotic is adequate in preseptal cellulitis.
d. ( ) Isolated orbital complication is commonest in children.
e. ( ) Subdural abscess is the commonest intracranial
complication.

3. Inverted papilloma,
a. ( ) Convoluted cribriform pattern is seen in Magnetic
Resonance Imaging scan.
b. ( ) Is more common in females.
c. ( ) Medial maxillectomy is the treatment of choice in Krouse
Staging III.
d. ( ) Postulated to have viral aetiology.
e. ( ) Risk of malignant transformation is up to 30%.

4. Classical CT scan findings in juvenile naso-angiofibroma include


a. ( ) erosion of hard palate
b. ( ) Holman-Miller sign
c. ( ) homogenous mass
d. ( ) obliteration of superior orbital fissure
e. ( ) widening of pterygopalatine fossa

5. Regarding rhinitis
a. ( ) Angiotensin Converting Enzyme inhibitor is a cause.
b. ( ) Immunotherapy is recommended for mild intermittent
allergic rhinitis.
c. ( ) Intermittent rhinitis symptoms occur more than 4 days per
week.
d. ( ) Nasal blockage occurs 10 minutes after mast cell
degranulation.
e. ( ) Non-allergic rhinitis is associated with asthma.

6. Regarding septal perforation


a. ( ) Biopsy is routinely done.
b. ( ) Bipedicle flaps are used for small defect repair.
c. ( ) It is a complication of intranasal corticosteroids spray.
d. ( ) The use of nasal splints during septal surgery is a
contributing factor.
e. ( ) Urine analysis is one of the investigations.

7. Regarding cerebrospinal fluid rhinorrhea


a. ( ) Computerized tomography cisternography is
recommended.
b. ( ) Endoscopic septoplasty is a cause.
c. ( ) Herniation of brain at fractured site is a reason for delayed
type.
d. ( ) Middle turbinate flap is used for cribriform plate repair.
e. ( ) Previous spinal surgery is a contraindication for fluorescein
lumbar puncture.
8. Regarding sinonasal fungal ball
a. ( ) Aspergillus sp. is the cause.
b. ( ) Calcification seen on computed tomography scan of
paranasal sinus is diagnostic.
c. ( ) Intravenous amphotericin is indicated.
d. ( ) It has a predilection for ethmoid sinus.
e. ( ) Persistent cacosmia is a presentation.

9. Regarding atrophic rhinitis


a. ( ) Closure of nostril is the treatment.
b. ( ) Halitosis is a presentation.
c. ( ) Its causative organism is Klebsiella ozaenae.
d. ( ) Microscopically seen as loss of ciliated columnar
epithelium.
e. ( ) Saddle nose deformity is a complication.

10. Sinonasal plasmacytoma


a. ( ) accounts for 60% of plasma cell neoplasms.
b. ( ) has a male preponderance.
c. ( ) involves the mucosa-associated lymphoma tissue (MALT) of
the upper airways in 10% of the cases.
d. ( ) is primarily treated with surgery.
e. ( ) leads to multiple myeloma.
11. Calcification or ossification on CT imaging is seen in
a. ( ) chondrosarcoma.
b. ( ) fibrous dysplasia.
c. ( ) osteogenic sarcoma.
d. ( ) osteoma.
e. ( ) squamous cell carcinoma.

12. Regarding frontal sinus drainage pathway


a. ( ) One of the drainage patterns of the frontal sinus is via
ethmoido-maxillary route.
b. ( ) Terminal recess is formed when the uncinate process is
attached to the skull base.
c. ( ) The drainage is obstructed in the presence of a retrobullar
cell.
d. ( ) The presence of agger nasi cell obstructs the drainage
pathway.
e. ( ) The superior compartment is formed by the union of
adjacent air cells at the postero-inferior portion of the ethmoid
bone.

13. Assessment of nasal airflow:


a. ( ) Cottle manouvre is positive if the nasal obstruction
disappears with retraction of the alar rim.
b. ( ) Acoustic rhinometry evaluates structural causes of nasal
obstruction.
c. ( ) Objective assessment of nasal air flow includes a SNOT-22
questionnaire.
d. ( ) Peak nasal inspiratory flow is an objective test.
e. ( ) The external nasal valve forms the narrowest part of the
nasal cavity.
May 2019
1. Rhinosporidiosis:
a. ( ) caused by Klebsiella F – fungal
b. ( ) Presents with epistaksis T
c. ( ) Source of infection contaminated water of ponds T
d. ( ) excision with cauterization of mass T

2. Secondary mediators of inflammation in Allergic rhinitis:


a. ( ) Histamine
b. ( ) Leukotriene
c. ( ) Bradykinin
d. ( ) Chemokines

3. Lupus vulgaris( TB nose)


a. ( ) Painless T
b. ( ) Apple jelly nodules T
Nov 2019
1. Plasmacytoma
a. ( ) Main treatment by surgery
b. ( ) Sequelae is multiple myeloma
c. ( ) Common plasma cell neoplasm
d. ( ) Arises from lateral wall of nose
e. ( ) MALT

2. Septal perforation
a. ( ) Posterior in TB
b. ( ) Wegeners involving whole septum
c. ( ) Rhinosporiodosis
d. ( ) Advancement flap for small perforation
e. ( ) Temporalis fascia to close medium sized perforation

3. Inverted papilloma
a. ( ) Common in females
b. ( ) Cribriform in MRI
c. ( ) Krause 3 medial maxillectomy
d. ( ) HPV 6
e. ( ) Malignancy in 30% in 5 years

4. Granulomatous disease markers


a. ( ) Sarcoidosis SMA
b. ( ) Wegeners pANCA
c. ( ) Lymphoma LDH
d. ( ) Sjogren syndrome A.S antibody
e. ( ) TB nucleic acid assay

5. Radiological signs of JNA


a. ( ) Holman miller
b. ( ) Hondusa
c. ( ) Widening pterygomaxillary fossa
d. ( ) Unilateral homogenous
e. ( ) Erosion of hard palate

6. Treatment of AR/CRS
a. ( ) Amtihistamine causes immediate decongestion
b. ( ) Nasal corticosteroid in pregnancy
c. ( ) Nasal corticosteroid reduce inflammatory cells
d. ( ) Immunotherapy suppresses IgE formation
e. ( ) Leukotriene inhibits 5 lipo oxygenase

7. NAR
a. ( ) NARES has 50% eosinophils for diagnosis
b. ( ) High Cincinnati irritant index indicates mixed rhinitis
c. ( ) Capsaicin first NSAIDs
d. ( ) Pregnancy induced best treated with corticosteroids

8. Complications of FESS
a. ( ) Epiphora
b. ( ) PEA
c. ( ) Lateral rectus palsy
d. ( ) Atrophic rhinitis
e. ( ) Septal perforation
9. Smell
a. ( ) Dysosmia
b. ( ) Parosmia
c. ( ) Hyposmia
d. ( ) Anosmia
e. ( ) Cacosmia

10. Maxillectomy
a. ( ) Osteotomy at frontoethmoidal suture
b. ( ) IMAX bleeding is a sequelae
c. ( ) Pterygoid plate release is done early
d. ( ) Palatal involvement require Lip split

11. Orbital complications


a. ( ) Isolated orbital complications is Commonest in paeds
b. ( ) Lateral can be treated with endoscopic
c. ( ) CT scan mandatory in all cases with complications
d. ( ) Oral Abx is adequate in preseptal cellulitis
NOV 2020
1. EBV in NPC
a. It is an RNA
b. Genome is used for diagnosis
c. Present in saliva
d. Use for treatment monitoring
e. Related to Type 1 NPC

2. Cavernous sinus thrombosis


a. V3 involved
b. On CT scan engorgement of superior ophthalmic vein is seen
c. Sphenoid sinusitis most common cause
d. Commonest organism is streptococcus

3. Orbital complication / orbital cellulitis?


a. Proptosis is due to dehiscence of lamina papyracea
b. Orbital cellulitis is adequately diagnosed by clinical examination alone
c. Subperiosteal abscess drained endoscopically
d. More common in children

4. Antrochoanal polyp
a. Also known as Killian’s polyp
b. Common in young adult
c. Infection is one of the cause
d. Mucopolysaccharide changes on ground substance is one of pathogenesis
e. Rhinolalia is a presenting symptom

5. Juvenile nasoangiofibroma
a. MRI salt and pepper appearance
b. Homogenous in ct contrast enhanced? --- got ah?
c. Arise from sphenopalatine foramen
d. Erosion of pterygoid body is a feature
e. Endoscopic treatment in Kadish C

6. Mucocele
a. Heterogenous lesion on CT
b. Bone resorbing cytokines at epithelial
c. Opthalmological symptoms more common than neurological
d. Mucopolysaccharide change is one of theory
e. High protein content

7. Inverted papilloma
a. Occur at septum
b. Treatment is complete resection
c. Invagination of keratinizing squamous epithelium into stroma with intact
basement membrane
d. 30 percent risk of malignancy after 15 years
e. Endoscopic medial maxillectomy gold standard for...
8. Sphenopalatine artery ligation
a. Crista ethmoidalis is located anterior to posterior end of middle turbinate
b. Posterior septal branch of sphenopalatine artery run superior to sphenoid
sinus ostium
c. Mma is an approach

9. Adult epistaxis
a. Embolization is successful in 10% of cases
b. Bipp packing causes septal perforation
c. Posterior packing is usually required

10. Acute rhinosinusitis


a. Antihistamines decreases the symptoms
b. Double sickening sign suggestive of bacterial infection
c. Is viral in origin
d. Swab for culture is mandatory in all cases

11. Allergic rhinitis


a. Nasal provocation test is gold standard
b. Leukotriene inhibitor is second line treatment
c. Immunotherapy is indicated for SPT negative patient
d. Nasal turbinate mucosa IgE is more specific than Serum igE

12. Fungal sinusitis


a. Chronic invasive fungal sinusitis affects immunocompetent patient
b. Fungal ball requires topical antifungal

13. Frontal sinusotomy


a. Draf 3 removes superior septum
b. Draf 2b removes frontal sinus floor
c. Complete Uncinectomy is done in Draf 1
d. Draf 2a is similar to nasofrontal type 2 approach
ADDITIONAL SET
RHINOLOGY AND ANTERIOR SKULL BASE
1. The following are true of allergic rhinitis
a. Intradermal skin test is more sensitive than percutaneous skin
prick test
b. In all allergic rhinitis patient total serum Ig E is comparatively
higher than normal population
c. RAST (radioallergosorbent test) is an in vivo method of
measuring allergen specific IgE
d. Sodium chromoglycate is effective only as a prophylactic agent
e. Intranasal corticosteroid is a treatment of choice because of
superior afficacy and safety

2. The following are true of choanal atresia


a. Results from a persistent nasobuccal membrane
b. About 80% are membranous
c. Unilateral in two-thirds of the cases
d. When bilateral presents with periodic apnoea in between crying
spells
e. Is associated with CHARGE syndrome

3. The following statement are true concerning a 16 years old girl presenting
to your clinic with a history of epistaxis , nasal obstruction and middle ear
effusion
a. Allergic polyposis
b. Inverted papilloma
c. Juvenile nasopharyngeal angiofibroma
d. Thornwald’s cyst
e. Nasopharyngeal carcinoma

4. The following statement are true of the anatomic indentation involving


the sphenoid sinus
a. Vidian nerve
b. Optic nerve
c. Frontal branch of trigeminal nerve
d. Maxillary branch of trigeminal nerve
e. Internal carotid artery

5. Indication for Caldwell- Luc procedure include


a. Antrochoanal polyp
b. Concha bullosa mucocele
c. Fungal maxillary sinusitis
d. Maxillary mucocele
e. Paradoxical middle turbinate

6. The following are true with regards to the arterial branches supplying the
Little’s area:
a. Septal branch of facial artery
b. Anterior ethmoidal artery
c. Greater palatine artery
d. Posterior nasal artery
e. Sphenopalatine artery
7. Paranasal sinus tumours associated with calcification or ossification
include
a. Fibrous dysplasia
b. Chondrocarcinoma
c. Squamous cell carcinoma
d. Osteoma
e. Osteogenic sarcoma

8. Subdural haematoma, brain tumour and meningitis are likely to have the
following in association except
a. Neck stiffness
b. Papilloedema
c. Fever
d. Visual aura
e. Vomiting

9. Which of the following is included in the differential diagnosis of nasal


crusting lesions
a. Syphilis
b. Churg-Strauss syndrome
c. Wegener’s granulomatosis
d. Midline lethal granuloma
e. Eosinophilic non allergic rhinitis

10. Skin test result are affected by


a. Recent allergen exposure
b. Age
c. Time of day of testing
d. Antihistamines
e. Corticosteroids

11. Unilateral purulent rhinitis in a 8 year old boy is most likely due to
a. Cystic fibrosis
b. Juvenile nasopharyngeal angiofibroma
c. Foreign body
d. Allergy
e. Unilateral choanal atresia

12. Juvenile nasopharyngeal angiofibroma


a. Is a most common in adolescent boys
b. Occasionally metastasizes
c. Has been linked to endocrine abnormalities
d. Is most common in pre-adolescent boys
e. Is best managed with radiation therapy

13. The following are true of paranasal sinus tumours


a. Adenocarcinoma of the ethmoids is more common in wood
workers
b. There is an increased incidence of squamous carcinoma among
heavy metal workers
c. The commonest malignant tumour is lymphoma
d. Retropharyngeal node metastasis in malignant tumours is
common
e. An ameloblastoma is a rapidly growing malignant tumour
14. The following are true of rhinolith
a. It is concretion of calcium salts around a foreign body in the
nasal fossa
b. It is characterized by pain
c. It gives a grating sensation on probing
d. It is radio-opaque
e. It can cause palatal erosion

15. The following are true of osteomyelitis of the frontal bone


a. It is frequently due to acute frontal sinusitis in children
b. Streptococci and staphylococci are the common bacteria
responsible
c. Surgery is indicated in the presence of sequestrum
d. It can ultimately result in Pott;s Puffy tumour
e. Long- term parenteral antibiotic are indicated

16. The following are true of inverted papilloma of the nose


a. It arises from the lateral nasal wall
b. Histopathologically the epithelium is grossly thickened and
infolded with common epithelial atypia
c. Malignant transformation occurs in 5% to 8% of cases
d. Radiotherapy is the treatment of choice
e. It is mainly unilateral with a male preponderance

17. The following are true with regards to atrophic rhinitis


a. The patients suffers from cacosmia
b. The primary type is commonest in young women
c. Hensen’s disease is a differential diagnosis
d. Local treatment is with 25% glucose in glycerine drops
e. Young’s operation is the surgical treatment of choice

18. The following statement are true of nasal anatomy


a. The cell bodies of olfactory neurons lie in the nasal mucosa
b. The general sensory supply of most of the inferior turbinate is by
the greater palatine nerve
c. The posterior lateral nasal nerve are branches of the superior
ethmoidal nerve
d. Lymph from the anterior part of the nose drains to the
submental nodes
e. The lymphatic drainage from the posterior part of the nasal
cavity is to the retropharyngeal and upper deep cervical lymph
nodes

19. Fungal sinusitis


a. Is suspected when symptoms fail to respond to standard
antibiotic therapy
b. Is common in immun0-compromised patients
c. Rarely caused by candida species
d. Is primarily treated by systemic antifungal agents
e. On CT shows a focal hyperdense lesion surrounded by
hypodense material

20. The frontal sinus


a. Develops from anterior portion of nasal capsule
b. Is demonstrated radiologically after 3 years of age
c. Opens through the frontal recess
d. Has an anterior plate made up of compact bone
e. Has a development failure on one side in 40% of the population
21. The following are true of the disease of the nasal septum
a. Syphilis involves perforation of the quadrilateral cartilage
b. Pyogenic granuloma is associated with pregnancy
c. Bleeding polyps are allergic in origin
d. Hereditary telangiectasia is treated by septodermoplasty
e. In septal hematoma resorption of the cartilage is due to pressure
necrosis

RHINOLOGY
1 TFFTT
2 TFTTT
3 FFFFT
4 TTFTT
5 T_TTF

6 TTTFT
7 FFTFF
8 FFFTF
9 TTTTF
10 FTTTF
11 FFTFT
12 TFTFF
13 TTFFF
14 TFTTT
15 FTTTT
16 TFTFT
17 FTTTT
18 TTFFT
19 TTTFT
20 TFTFF
21 FTFTF
USM COLLECTION

1. The following is/are nasal manifestation of AIDS


a. Kaposi’s sarcoma
b. Seborrheic dermatitis
c. Candidiasis
d. Lymphoma
e. Rhinosinusitis
TTTTT

2. Regarding the physiology of nose and PNS


a. Internal nasal valve contributes to 50% of total nasal resistance
b. Pungent odors may be perceival even in olfactory nerve damage
c. The mucous blanket consist of superficial serous layer and
deeper mucous layer
d. The mucous blanket moves at speed of 5-10mm per minute
e. The presence of nasal packing can cause lowering of pO2
TTFTT

3. Osler-Weber-Rendu syndrome
a. Is an autosomal recessive condition
b. Lesions often occurs at birth
c. The defect is in the contractile elements of vessel
d. Bleeding is difficult to control as the muscular coating is lacking
e. The heterozygotes state is lethal at an early age
FFTTF
LARYNX
NOV 2007
1. In total laryngectomy
a. Thyroid excised at the ipsilateral side only
b. Check for thyroid hormone next morning post op
c. Check ca level next morning
d. Associated with thyroid storm
e. With stomach pull up procedure there is hypercalcemia
FFTFF

2. Trachea (vocal cord)


a. Reinke’s oedema is associated with smoking
b. Reinke’s oedema treated by voice therapy
c. Squamous cell carcinoma is common in the posterior
commissure
d. Surgery of vocal cord can end with vocal cord scar
TTFT

3. Vocal cord
a. Vocal cord granulomas are common
b. Vocal cord cysts occur mainly at the junction of the anterior 2/3
and posterior 1/3
c. Vocal cord nodule are common at the anterior third
FFT

4. Regarding the larynx


a. Recurrent laryngeal nerve palsy is ipsilateral when the cause at
the level of the midbrain
F

5. Laryngocele
a. Bryce’s sign is one of the features
b. Common in female
c. It is occupational disease
TFT
MAY 2008
1. Vocal cord palsy
a. Granuloma occurs in type 1 thyroplasty
b. Arythenoid adduction is to correct posterior commissure gap
c. Gelfoam is use to correct a temporary symptom
FTT

2. Benign laryngeal lesion


a. Vocal nodule is subepithelial
b. Vocal nodule never involved deep layer of lamina propria
c. Vocal polyp can be treated by speech therapy
d. Single papilloma can become malignant
e. Speech therapy is the choice for singer’s nodule
FTFTT

3. In ca larynx
a. Glottic is the commonest subgroup
b. In supraglottic malignancy pre epiglottic space is involve
c. If anterior commissure is involve thyroid cartilage is involved
d. In transglottic ca, para glottic space is involve
TTTT

4. In laryngectomy
a. A pre-epiglottic space is not entered
b. Total thyroidectomy is advocated in laryngectomy
TT

5. RRP
a. Transplacenta transmission can occur
b. If first child borne has RRP, C-section is for the next delivery
c. CMV is a co-factor
d. Cidofovir is given intravenously
TFFF

6. In laryngeal trauma
a. Fracture of hyoid is not significant clinically
b. Tracheostomy is preferred t intubation
TT

7. In oesophageal speech
a. Taub test is mandatory
b. Cricopharyngeal myotomy is not to be performed
c. Vibration is due to pharyngo- oesophageal segment
d. Have to do TE fistula
e. Need good lung reserve
FFTFF

8. Benign laryngeal lesions


a. Sulcus vocalis is treated by cordectomy
b. Proton pump inhibitor has no role in intubation granuloma
FF
NOV 2008
1. Stomal recurrence after total laryngectomy is related to
a. A tracheostomy positioned close to the tumour
b. Subglottic extension
c. Thyroid gland involvement
d. Paratracheal node secondaries
e. Supraglottic carcinoma
TTTTF

2. Laryngeal intraepithelial neoplasia includes


a. Dyskeratosis
b. Parakeratosis
c. Verrucous carcinoma
d. Dysplasia
e. Carcinoma in situ
TTFTT

3. Videostroboscopic examination is for the assessment of


a. Mucosal wave
b. Glottal closure
c. Vocal cord scar
d. Vocal cord laxity
e. Vocal pitch
TTTTF

4. The cause of chronic laryngeal stenosis


a. Autoimmune thyroiditis
b. Polychondritis
c. Wegener’s Granulomatosis
d. Pharyngocutaneous fistula
e. Tuberculous laryngitis
TTTFT

5. Regarding unilateral vocal cord paralysis


a. The paralysed cord is fixed in cadaveric position
b. Branchial carcinoma is the most common cause
c. Right recurrent laryngeal nerve involvement is more than left
side
d. Treatment is expectant in the first six months
e. Presentation is a late manifestation of primary lesion
FFFTT

6. Laryngeal verrucous carcinoma


a. Occurs commonly in the larynx
b. In the larynx occurs predominantly in supraglottis
c. Is treated primarily by radiotherapy
d. Infiltrates on the broad base with pushing margins
e. Rarely metastases to the lymph nodes
FFFTT

7. Which type of laser cuts tissue by vaporizing intercellular water?


a. KTP
b. CO2
c. Nd: YAG
d. Diode
e. Argon
FTFFF
8. Hyperkeratosis of the larynx
a. This is premalignant lesion
b. Should be treated with chemotherapy
c. It produces impairment of vocal cord mobility
d. Associated with HPV
e. Proton pump inhibitor is acceptable treatment
TFFFF

9. Regarding recurrent respiratory papillomatosis


a. It is a primary disease of neonates
b. Radiotherapy is treatment of choice
c. Early tracheostomy is required
d. HPV type 11 is more aggressive than type 6
e. Cidofovir can be injected locally.
FFFTT

10. With regards to intubate injuries


a. Intubation granulomas are caused by pressure by the ETT on the
muscular process of the arytenoid cartilage
b. Diameter, shape, and contour of the ETT influences the
incidence of laryngeal injury
c. Laryngopharyngeal reflux increase the incidence of laryngeal
injury
d. Ulcerated through occur in the midline of the posterior glottis
e. Cricoarythenoid joint is susceptible to injury
FTTFT

11. The following statements regarding tracheal injury are true


a. The degree of emphysema is proportional to the size of tracheal
perforation
b. Injury of the recurrent laryngeal nerve is an indication for
exploration
c. When indicated , tracheostomy is best done away from the site
of injury
d. Tracheal stenosis is a common sequalae
e. Laryngomalacia is more likely when the injury is complicated by
infection
FFFTT

12. The following are true of tracheostomy in adults


a. A high tracheostomy should be performed for stridor secondary
to a laryngeal tumour
b. Massive surgical emphysema of the head and neck is an
indication
c. Complication include injury to the brachio cephalic artery
d. Tracheostomy is an initial treatment for uncontrolled aspiration
e. It is often necessary in adult acute epiglottitis
TTTTF

13. Laryngopharyngeal reflux


a. Is aggravated by consumption of caffeine
b. Gives rise to various grades of dysphonia
c. Is diagnosed primarily by manometric studies
d. Is initially treated with motility enhancing medication
e. Increase the risk of stenosis inpatient who undergo prolonged
intubation
TTFFT
May 2013
1. TB larynx
a. ( ) Anterior commissure commonly affected
b. ( ) Ulcer in vocal cord
c. ( ) Epiglottic oedema due to perichondritis
d. ( ) Presents of verrucous mass at supraglottis
e. ( ) Clinically distinguishable from carcinoma

2. Vocal polyp
a. ( ) >3mm
b. ( ) Response to voice therapy
c. ( ) Bilateral
d. ( ) Male > female
e. ..

3. Causes of poor quality voice post-thyroplasty type 1


a. ( ) Post op oedema
b. ( ) Vocal cord scarring
c. ( ) Implant migration
d. ( ) Inadequate arytenoid adduction
e. ( ) Improper placement of implant

4. …………..
a. ( ) Cricoarythenoid dislocation diagnosed intra op
b. ( ) Arythenoid adduction good for large phonatory gap

5. True regarding glottic ca


a. ( ) Paraglottic extension indicates advanced disease
b. ( ) Metastasis to the submandibular gland
c. ..
d. ..
e. ..

6. Regarding microsurgery in the larynx


a. ( ) Saline injection is beneficial in micro flap elevation
b. ( ) Adrenaline injection is relatively contraindicated in VC
polyp
c. ( ) Arytenoid fixation is confirmed intraoperatively
d. ( ) Micro flap elevation in Reinke’s oedema
e. ( ) Kee can be inserted endoscopically
May 2014
1. Which are the cause of stomal stenosis
a. ( ) Tumour recurrence
b. ( ) Inadequate removal of peri-stoma fat
c. ( ) Use of monofilament suture
d. ( ) Use of stomal button
e. ( ) Primary trachea-esophageal puncture with prosthesis

2. Contra-indication of supraglottic laryngectomy


a. ( ) Poor lung function
b. ( ) Involvement of vocal cord
c. ( ) Reduced mobility of vocal cord
d. ( ) Involvement of preglottic area
e. ( ) Involvement of paraglottic

3. LASER
a. ( ) Used of pulsed mode can minimized risk of fire
b. ( ) Is best use in benign laryngeal lesion
c. ( ) Nd- YAG can be delivered by fibreoptic
d. ( ) Less lateral thermal damage
e. ( ) High oxygen is important in reducing the risk of fire
complication

4. Reinke oedema
a. ( ) 40% are bilateral
b. ( ) The synonym is pseudomyxomatrix laryngitis
c. ( ) Associated with hypothyroidism
d. ( ) Has associated with malignancy transformation

5. Hyperkeratosis of larynx
a. ( ) Is a premalignant state
b. ( ) Limit vocal cord movement
c. ( ) It cause ulceration
d. ( ) It treated with radiotherapy
e. ( ) Presence of lymph node

6. Laryngocele
a. ( ) Is caused by glottic carcinoma
b. ( ) 90% is combined external -internal laryngocele
c. ( ) Can be approach by laryngofissure
d. ( ) Can be demonstrated by asking patient to do Valsalva
during xray
e. ( ) Can be seen in radiography

7. In cricoaryhtenoid subluxation, option of surgery


a. ( ) Endoscopic close reduction
b. ( ) Laryngofissure
c. ( ) Medialization thyroplasty
d. ( ) Injection laryngoplasty – vocal cord

8. Regarding laryngectomy
a. ( ) Preepiglottic fold entered before tumour resection
b. ( ) Greater horn of hyoid preserve
c. ( ) Paracarotid avoided
9. Treatment of intractable aspiration
a. ( ) Epiglotopexy
b. ( ) Closure of epiglottis
c. ( ) Partial laryngectomy
d. ( ) Type 1 thyroplasty
e. ( ) Cricomyotomy

10. Stroboscope able to assess


a. ( ) Vocal cord weakness
b. ( ) Frequency
c. ( ) Pitch
d. ( ) Mucosa wave
e. ( ) Amplitude of the wave
May 2015
1. Laryngeal trauma
a. ( ) Tracheostomy preferred then intubation
b. ( ) Surgical emphysema due to pneumothorax
c. ( ) If no bruises, need to rule out tracheal injury
d. ( ) Need to suspect spine injury

2. Reflux oesophagitis
a. ( ) MTD in LPR
b. ( ) Ventricle obliteration
c. ( ) 6 weeks on PPI shows improvement
d. ( ) Manometry is superior than 24h double probe

3. Regarding reflux pharyngeal pharyngitis


a. ( ) Pain is a symptom for reflux oesophagitis
b. ( ) Achalasia more fluid to solid?
c. ( ) Plummer-Vinson syndrome TBC is high

4. Stoma recurrence
a. ( ) Incidence 2-5%
b. ( ) Treatment sisson type III?
c. ( ) Cant be prevented by not doing tracheostomy
d. ( ) Tracheal lymph nodes involvement
e. ( ) Radiotherapy is the cause

5. Laryngotracheal reconstruction
a. ( ) Vocal cord palsy need double stage
b. ( ) Single stage surgery for post glottic stenosis
c. ( ) Grade IV Cotton Myer
d. ( ) Aspiration is relative contra-indication
e. ( ) Treatment consist of ?

6. Contra-indication of supraglottic laryngectomy


a.
b.

7. Videostroboscopy
a. ( ) Mucosal wave
b. ( ) Glottal wave
Nov 2015
1. Video stroboscopy to detect
a. ( ) Mucosal amplitude
b. ( ) Non mobile part of vocal fold
c. ( ) Mucosal wave
d. ( ) Glottal closure
e. ( ) Voice intensity

2. Absolute contraindicated for jet ventilation


a. ( ) Obesity
b. ( ) Emphysema
c. ( ) Tracheomalacia
d. ( ) Grade IV CM

3. Treatment of vocal nodule


a. ( ) Cordectomy type II
b. ( ) Treat with PPI
c. ( ) Vocal hygiene
d. ( ) Diaphragmatic breathing

4. Chronic aspiration
a. ( ) Treat with laryngeal suspension
b. ( ) Tracheostomy
c. ( ) Gastrostomy
d. ( ) Myotomy (upper oesophageal sphincter)
e. ( ) Laryngeal denervation

5. Verrucous larynx
a. ( ) EBV detected in tissue
b. ( ) Radiotherapy is more effective than SCC
c. ( ) Similar with leukoplakia
d. ( ) T3 indicated for total laryngectomy

6. Hyperkeratosis of larynx
a. ( ) Nibble appearance
b. ( ) VC mobility affected
c. ( ) Radiotherapy is treatment
d. ( ) Is premalignant lesion
MAY 2016
1. You are called to attend a patient on home tracheostomy that has come
in acutely breathless. Which of the following are possible causes of the
breathlessness?
a. ( ) Mucous pus within the tracheostomy tube
b. ( ) Granulation tissue at the tip of the tube
c. ( ) Tracheomalacia
d. ( ) Subglottic stenosis
e. ( ) Trachea-oesophageal fistula

2. Regarding lesion of the larynx


a. ( ) Reinke’s oedema is a premalignant condition
b. ( ) Subcordal oedema is also called sulcus vocalis
c. ( ) Contact granuloma is treated medically
d. ( ) Ventricular obliteration is a sign of LPR
e. ( ) Posterior commissure hypertrophy is secondary to
intubation

3. Unilateral vocal cord palsy


a. ( ) Hyaluronic acid can be used as a temporary procedure
b. ( ) Bowing is a sign of vocalis atrophy
c. ( ) Associated with muscle tension dysphonia
d. ( ) Sharp waves on EMG is a sign of reinnervation
e. ( ) Posterior cervical spine instrumentation is a cause

4. Laryngeal trauma
a. ( ) Schaeffer II require surgery
b. ( ) Best to intubate
c. ( ) CT first line investigation
d. ( ) Hyoid bone has no clinical significant

5. Reinke’s oedema
a. ( ) Chronic renal failure
b. ( ) Tobacco implantation
c. ( ) Hyperthyroidism
d. ( ) Voice strain
e. ( ) Reflux
NOV 2016

1. The following about oesophageal speech


a. ( ) It is produced by vibration of PES
b. ( ) It required creation of tracheoesophageal fistula
c. ( ) Cricopharyngeal myotomy precludes for oesophageal
speech
d. ( ) It is steep learning curve
e. ( ) Taub test is performed as prerequisite

2. Risk factor of developing laryngeal Squamous malignancy


a. ( ) Supraglottic ca more common in women compare to male
b. ( ) Laryngeal cancer is common in areas with higher level of
Social deprivation
c. ( ) Reflux disease carries major risk
d. ( ) Alcohol promotes carcinogenesis through acetyldehyde
exposure
e. ( ) Epstein Barr virus infection increase the risk

3. Regarding verrucous ca of larynx


a. ( ) Is more radiosensitive then SCC
b. ( ) T4 disease is treated with total laryngectomy
c. ( ) It is poorly differentiated SCC
d. ( ) Smoking is risk factor
e. ( ) Exhibits positivity for Epstein Bar virus

4. Tracheomalacia
a. ( ) Generalized form is more common than localized
b. ( ) Collapsed at its anteroposterior diameter
c. ( ) Caused by compression of innominate artery
d. ( ) Presented with expiratory stridor
e. ( ) Complete spontaneous recovery does not occur

5. Contraindication of open surgical approach for Laryngotracheal stenosis


a. ( ) Bilateral abductor vocal fold paralysis
b. ( ) Severe aspiration
c. ( ) Poor lung reserve
d. ( ) Posterior glottic stenosis
e. ( ) Infected tracheostomy

6. Regarding laryngopharyngeal reflux


a. ( ) Absence of acid reflux excludes the diagnosis
b. ( ) Thick endolaryngeal mucous is one of the signs
c. ( ) Poor response to medical treatment require OGDS
d. ( ) Fundoplication is a surgical option
e. ( ) Treatment with antacids prevent occurrence of reflux

7. Laryngeal causes of chronic cough include


a. ( ) Papillomatosis
b. ( ) Reflux disease
c. ( ) Tuberculosis
d. ( ) Relapsing polychondritis
e. ( ) Amyloidosis
8. Objective evaluation of voice includes
a. ( ) Aerodynamic measurement
b. ( ) Perceptual evaluation
c. ( ) Voice handicap index
d. ( ) Acoustic measurement
e. ( ) Voice symptom score

9. Bilateral abductor palsy of the vocal cords


a. ( ) Is commonly caused by malignant thyroid tumours
b. ( ) Presents with breathy voice in chronic cases
c. ( ) Is diagnosed by examination under anaesthesia
d. ( ) Is treated by Teflon injection
e. ( ) Tracheostomy is the suggested treatment for professional
voice users
Nov 2017
1) Essential tremor of voice
a. ( ) Paradoxical vocal motion
b. ( ) Adduction during inspiration
c. ( ) Associated with LPR
d. ( ) Tracheostomy is the treatment
e. ( ) Anxiolysis is effective

2) Laryngocele
a. ( ) Can communicate through CT space
b. ( ) Fluctuating in size
c. ( ) Commonly presents as midline swelling
d. ( ) USG is best form of imaging
e. ( ) Surgical excision complicated with injury to RLN

3) Causes of incomplete glottic closure


a. ( ) Unilateral VC palsy
b. ( ) Vocal cord nodule
c. ( ) Vocal cord cyst
d. ( ) Abductor spasmodic dysphonia
e. ( ) Presbyphonia

4) Symptoms of LPR
a. ( ) Productive cough
b. ( ) Hoarseness
c. ( ) Chest pain
d. ( ) Frequent throat clearing
e. ( ) Dyspnoea to pills

5) Findings in TB
a. ( ) Turban epiglottis
b. ( ) VC immobility
c. ( ) VC bowing
d. ( ) Interarytenoid granulation tissue
e. ( ) Cobblestone mucosa

6) PORT in Ca Larynx
a. ( ) Microscopically positive margin
b. ( ) Positive node in 1st echelon nodes
c. ( ) Perineural invasion in post op specimen
d. ( ) T4 tumour
e. ( ) Lymph node size more than 2cm
NOV 2018
1. Conditions causing incomplete glottal closure include
a. ( ) sulcus vergeture
b. ( ) presbyphonia
c. ( ) chronic laryngitis
d. ( ) abductor spasmodic dysphonia
e. ( ) pubophonia

2. Spasmodic dysphonia
a. ( ) Electromyography improves localization of the
thyroarytenoid muscle.
b. ( ) Laser thyro-arytenoid myoneurectomy is a surgical option
in adductor type.
c. ( ) Magnetic Resonance Imaging shows white matter changes
in the thalamus.
d. ( ) The posterior cricoarytenoid muscle is spared.
e. ( ) Unilateral botulinum toxin injection is as efficaceous as
bilateral injections.

3. Regarding complications of tracheostomy


a. ( ) aerophagy is treated with deflation.
b. ( ) mediastinitis occurs following peristomal wound infection.
c. ( ) pneumothorax is commoner in children compared to
adults.
d. ( ) the common carotid artery is implicated in late
haemorrhage.
e. ( ) ventricular fibrillation occurs during tracheal suctioning.

4. Biomaterials used for medialization laryngoplasty,


a. ( ) autologous fat injection provides predictable duration of
effect.
b. ( ) calcium hydroxyapatite is indicated in lamina propria
injection.
c. ( ) expanded polytetrafluoroethylene is used in Type 1
thyroplasty.
d. ( ) hyaluronic acid injection is a fast method for temporary
medialization of the vocal fold.
e. ( ) prefabricated titanium implants demands less surgical time
compared to silastic implants.

5. Laryngeal tuberculosis,
a. ( ) failure to improve following anti-tuberculosis mandates a
biopsy.
b. ( ) laryngocele is a sequealae.
c. ( ) low ESR excludes the diagnosis.
d. ( ) the differential diagnosis is laryngeal carcinoma.
e. ( ) the lesion is commonly at the anterior commissure.
6. Regarding laryngeal carcinoma
a. ( ) Carcinoma in situ is managed with endoscopic excision.
b. ( ) Glottic carcinoma which extends to subglottis is considered
T3.
c. ( ) Laryngopharyngeal reflux is a risk factor.
d. ( ) Minor salivary gland carcinoma is the commonest type.
e. ( ) Most common site is supraglottis.

7. Regarding laryngeal trauma


a. ( ) Disruption of the anterior commissure is an indication for
early surgical repair.
b. ( ) Endoscopic examination shows mucosal hematoma.
c. ( ) Laryngeal trauma with exposed cartilage is class II Schaefer.
d. ( ) Presence of biphasic stridor suggests subglottic
involvement.
e. ( ) Recurrent laryngeal nerve injury is a complication of
prolonged endotracheal tube ventilation.
Nov 2019
1. Laryngeal carcinoma poor prognosis
a. ( ) DNA aneuploidi
b. ( ) Involve anterior commissure
c. ( ) HPV positive
d. ( ) VC immobility
e. ( ) Presence of Paratracheal nodes

2. LPR
a. ( ) Alginate helps non acidic reflux
b. ( ) PPI is not effective in P450 cytochrome problems
c. ( ) PPI Partially inhibits H2 in parietal cells
d. ( ) Nissen fundoplication in intractable cases

3. Larynx
a. ( ) Intermediate and deep LP makes up vocal ligament
b. ( ) Reinke space is highly vascularised
c. ( ) Arcuate line divides glottic and subglottic
d. ( ) Vibration of VC is vertically from superior to inferior

4. Ca larynx
a. ( ) Minimal involvement of inner cortex is T3
b. ( ) Presence of aspiration is contraindicated for laryngeal
preservation surgery
c. ( ) Sclerosis of arytenoid cartilage is contraindicated for laser
surgery

5. Spasmodic dysphonia
a. ( ) Abductor type is more common than adductor type
b. ( ) Speech therapy is a treatment
c. ( ) Associated with vocal tremor
d. ( ) Abductor is also known as plica ventricularis
e. ( ) Commonly in females

6. Objective assessment of voice


a. ( ) Acoustic analysis
b. ( ) Aerodynamic measurement
c. ( ) Voice perception
d. ( ) Self reporting outcome
e. ( ) Voice score system
NOV 2020

1. Objective measurement of voice


a. Acoustic analysis
b. Self Evaluation Questionaire
c. Perceptual analysis
d. Voice handicap index
e. Aerodynamic

2. Regarding cricotracheal resection


a. Indicated in CM Grade IV
b. Lateral resection posterior to cricoarytenoid joint
c. Contraindicated in posterior cricoid split
d. One stage surgery in posterior glottic stenosis
e. Result in posterior suture granuloma if done posterior

3. Regarding verrucous carcinoma of larynx


a. Locally invasive
b. Low grade
c. Treated by radiotherapy
d. It is called Ackerman tumour
e. distant metastasis is uncommon

4. Laryngeal granulomatous disease


a. Turban epiglottis in tuberculosis
b. Mikulizch cells seen in scleroma
c. Punched-out lesion in syphilis
d. Ace increases in wegener
e. Non caseating granuloma in sarcoidosis

5. Total laryngectomy
a. Total thyroidectomy routinely done in all cases
b. Paracarotid tunnel is done after separation of infrahyoid muscle
c. Cricopharyngeal myotomy is done until mucosa

6. Injection laryngoplasty
a. Fat is an option
b. TransThyroid injection no need LA into larynx
c. Injection to reinke space for medialisation
d. Teflon known to cause granuloma
e. Hyaluronic acid for short term

7. Type ii medialisation
a. Preformed silastic used to tense the vocal cord
b. Shortening of anterior commissure is one of the complication
c. Median thyrotomy required

8. Spasmodic dysphonia
a. Injection to thyroarytenoid muscle in adductor
b. Abductor injection to lateral cricoarytenoid muscle
c. Botox A is used
d. Usage of EMG is recommended
9. Cricopharyngeal myotomy
a. Incision up to mucosa
b. Submucosal nerve plexus is denervated
c. Precludes for TEP later
d. Cause dysphagia
e. Improved voice rehabilitation outcome

10. Laryngeal trauma


a. Cricoid fracture is commonest
b. Cricotracheal separation causes bilateral RLN injury
c. Loss of Laryngeal crepitus is a symptom
d. Neuronotmesis will have complete recovery
ADDITIONAL SET LARYNX
1. In acute epiglottitis
a. Soft tissue neck x-ray is a necessity for diagnosis
b. Dysphagia is the predominant symptoms in adults
c. The child is typically restless
d. Air leak around the endotracheal tube is a good indication for
extubation
e. Streptococcus is the commonest causative organism in adults

2. The following are true of congenital laryngeal anomaly


a. In laryngomalacia girls are affected more than boys
b. Dilatation is the treatment of choice for subglottic stenosis
c. Unilateral vocal cord paralysis often resolves in the first few
weeks of life
d. The incidence of laryngeal and tracheal cleft is less than 1 %
e. G syndrome is a known syndrome associated with laryngeal and
tracheal clefts

3. The following are true of relapsing polychondritis


a. It only involves the elastic cartilage
b. Positive ANCA test is diagnostic of the condition
c. Presents with polybeak nasal deformity
d. Mortality is due to involvement of the laryngeal cartilage
e. Cytotoxic drugs are used in its treatment

4. Acute epiglottitis
a. Is characterised by high grade fever
b. Is commoner in children than in adults
c. Is an infection confined to the epiglottis
d. Should be diagnosed by indirect laryngoscopy
e. Is treated primarily with steroid

5. The following are true of glottic carcinoma


a. It is highly curable in the early stages
b. Tumours involving the anterior commissure show a tendency
too spread to the hyoid cartilage
c. Stripping of the vocal cords is ideal treatment for carcinoma-in-
situ
d. Cancer persisting in the mucous gland is a cause of radiation
failure
e. Laser cordectomy has the advantage of producing hoarse voice

LARYNGOLOGY
1 FTFTT
2 TFFTT
3 FFTTF
4 TTFFF
5 TTF_F

You might also like